AAFP Missed Questions

¡Supera tus tareas y exámenes ahora con Quizwiz!

Time Fram for Charcoal use

1 hour

H/o diabetes, MI, chest pain, CAD or Men > 45 and Women > 55: Preop Clearance

12 Lead EKG

CKD recommended BP

130/80

Size to Repair AAA

5.5 cm

Correlation between A1c and mean glucose

A rough guide for estimating average plasma glucose levels assumes that an 1cof 6.0% equals an average glucose level of 120 mg/dL. Each percentage point increase in 1c is equivalent to a 30-mg/dL rise in average glucose. An HbA1cof 7.0% is therefore roughly equivalent to an average glucose level of 150 mg/dL, and an HbA1c of 8.0% translates to an average glucose level of 180 mg/dL.

Which one of the following should be avoided when treating pain in the elderly? (check one) A. Fentanyl (Sublimaze) B. Hydrocodone C. Meperidine (Demerol) D. Morphine E. Oxycodone (OxyContin)

According to the Beers criteria, a list of drugs that should generally be avoided in older adults, meperidine should not be used in the elderly because its metabolite can accumulate and cause seizures. The other medications are not listed in the Beers criteria and are not contraindicated in the elderly

Which one of the following benzodiazepines has the shortest half-life? (check one) A. Flurazepam (Dalmane) B. Alprazolam (Xanax) C. Clorazepate (Tranxene) D. Diazepam (Valium) E. Clonazepam (Klonopin)

Alprazolam (Xanax) has a half-life of about 12 hours, versus 25 hours for clonazepam and 50 hours for flurazepam, clorazepate, and diazepam.

You see a 22-year-old female who sustained a right knee injury in a recent college soccer game.She is a defender and executed a sudden cutting maneuver. With her right foot planted and her ankle locked, she attempted to shift the position of her body to stop an oncoming ball and felt her knee pop. She has had a moderate amount of pain and swelling, which began within 2 hours of the injury, but she is most concerned about the loss of knee hyperextension. Which one of the following tests is most likely to be abnormal in this patient? (check one) A. Anterior drawer B. Lachman C. McMurray D. Pivot shift

Anterior cruciate ligament (ACL) tears occur more commonly in women than in men. The intensity of play is also a factor, with a much greater risk of ACL injuries occurring during games than during practices. The most accurate maneuver for detecting an ACL tear is the Lachman test (sensitivity 60%-100%, mean 84%), followed by the anterior drawer test (sensitivity 9%-93%, mean 62%) and the pivot shift test (sensitivity 27%-95%, mean 62%) (SOR C). McMurray's test is used to detect meniscal tears.

Severe Preeclampsia Requirements

BP > 160/110, Proteinuria > 5g/24h, Thrombocytopenia w/ Proteinuria < 100,000, Liver enzyme abnormalities, AMS

The most common cause of abnormal vaginal discharge in a sexually active 19-year-old female is (check one) A. Candida albicans B. Trichomonas vaginalis C. Staphylococcus D. Group B Streptococcus E. Bacterial vaginosis

Bacterial vaginosis (BV) is the most common cause of acute vaginitis, accounting for up to 50% of cases in some populations. It is usually caused by a shift in normal vaginal flora.

Cilastozol contraindication

CHF

CHADS

CHF, HTN, Age > 75, Diabetes, Stroke; 2+ score = anticoagulate

Supraventricular Tachycardia Tx

Carotid Massage/Adenosine --> Beta Blocker or Verapamil --> Flecainide

What helps with claudication

Cilastozol

Outpatient DVT Tx

Enoxaparin (Lovenox)

Acute Intersitial Nephritis

Eosinophiliuria, pyuria, hypersensitivity to nsaids

Neuroblastoma markers

HVA, VMA (same as Pheo)

Hemorrhoid Tx: Internal vs. External

INTERNAL - Rubber band ligation EXTERNAL, NON-THROMBOSED - Laxatives, Sitz Bath EXTERNAL, THROMBOSED - Incision and removal

Hypertensive Options in Pregnancy

IV Hydralazine, Labetalol, Oral Nifedipine

Latent TB Infection Tx

Isoniazid for 9 months

HTN Tx w/ Diabetes

Lisinopril

Diabetes drug that increases insulin sensitivity

Metformin

Weight Loss causing Diabetes Drugs

Metformin, Exenatide

Treatment for Rosacea

Metronidazole and other Abx

When to do Thyroid Scan

Nodule + Decreased TSH

Hypercalcemia Tx

Normal Saline + Lasix if in danger of fluid overload Can do Pamidronate after fluid is replenished

Hip Osteoarthritis Risk Factors

Obesity, Old Age, Hypothyroidism, Weight Bearing Sports, High Bone Mass

Femoral Anteversion Tx

Observation

Pros vs. Con of HRT

PROS - Decreased Osteoporosis, Colon CA risk, decreased hot flashes CONS - Increased embolus, breast CA

Primary Dysmenorrhea

Pain with menses without an identifiable cause. Tx with NSAIDs

When prescribing an inhaled corticosteroid for control of asthma, the risk of oral candidiasis can be decreased by: (check one) A. using a valved holding chamber B. limiting use of the inhaled corticosteroid to once daily C. adding nasal fluticasone propionate (Flonase) D. adding montelukast (Singulair) E. adding salmeterol (Serevent)

Pharyngeal and laryngeal side effects of inhaled corticosteroids include sore throat, coughing on inhalation of the medication, a weak or hoarse voice, and oral candidiasis. Rinsing the mouth after each administration of the medication and using a valved holding chamber when it is delivered with a metered-dose inhaler can minimize the risk of oral candidiasis.

Best initial managment of A-Fib

Rate control with beta blockers or calcium channel blockers

Bulimina Tx

SSRIs

OCD Tx

SSRIs

Croup Tx

Single dose of steroids

Hirsutism Tx

Spironolactone and OCPs

The recommended time to screen for gestational diabetes in asymptomatic women with no risk factors for this condition is: (check one) A. in the first trimester B. at 16-20 weeks gestation C. at 24-28 weeks gestation D. at 35-37 weeks gestation

The recommended time to screen for gestational diabetes is 24-28 weeks gestation. The patient may be given a 50-g oral glucose load followed by a glucose determination 1 hour later

Nausea and Vomiting of Pregnancy Tx

Vitamin B6 Doxylamine

A positive flexion abduction external rotation (FABER) test that elicits posterior pain indicates involvement of which joint? (check one) A. Sacroiliac B. Shoulder C. Ankle D. Wrist E. Knee

When the flexion abduction external rotation (FABER) test elicits pain posteriorly, it indicates sacroiliac involvement. Anterior pain indicates hip involvement.

Risk of what with dermatomyositis

cancer

Complication of Psuedotumor cerebri

visual loss

Tuberculosis cutoffs: 5, 10, 15

> 5 is positive in immunocomprimised >10 is positive in children, nursing homes, correctional facilities, homeless, immigrants, IV drug users, hospital workers, chronic illnes > 15 is positive in everyone else

A 48-year-old white female comes to see you because of abnormal vaginal bleeding. Her periods are lasting 3-5 days longer than usual, bleeding is heavier, and she has experienced some intermenstrual bleeding. Her physical examination is unremarkable, except for a parous cervix with dark blood at the os and in the vagina. She has no orthostatic hypotension, and her hemoglobin level is 11.5 g/dL. A pregnancy test is negative. Which one of the following is the most important next step in management? (check one) A. Laboratory tests to rule out thyroid dysfunction B. An endometrial biopsy C. Oral contraceptives, 4 times a day for 5-7 days D. Cyclic combination therapy with conjugated estrogens (Premarin) and medroxy-progesterone (Provera) each month E. Administration of a gonadotropin-releasing hormone analog such as leuprolide acetate (Eligard Lupron Depot)

A patient over the age of 35 who experiences abnormal vaginal bleeding must have an endometrial assessment to exclude endometrial hyperplasia or cancer. An endometrial biopsy is currently the preferred method for identifying endometrial disease. A laboratory evaluation for thyroid dysfunction or hemorrhagic diathesis is appropriate if no cancer is present on an endometrial biopsy and medical therapy fails to halt the bleeding. The other options listed can be used as medical therapy to control the bleeding once the histopathologic diagnosis has been made.

A 15-year-old white male is being evaluated after a fall down one flight of stairs. He was transported by the local rescue squad with his cervical spine immobilized. He walked briefly at the scene and did not lose consciousness. His only complaint is a mild, generalized headache. One episode of vomiting occurred shortly after the accident. No weakness or numbness has been noted. Vital signs, mental status, and neurologic findings are normal. Radiologic evaluation of the cervical spine is remarkable only for an air-fluid level in the sphenoid sinus. Which one of the following abnormalities is most likely to be associated with this radiologic finding? (check one) A. A basilar skull fracture B. An orbital floor fracture C. An epidural hematoma D. A zygomatic arch fracture E. A mandible fracture

A post-traumatic air-fluid level in the sphenoid sinus is associated with basilar skull fractures. This finding is frequently noted on cervical spine films. Orbital floor fractures may be associated with double vision, fluid in the maxillary sinus, an air-fluid level in the maxillary sinus, and diplopia. Epidural hematomas are more frequently associated with skull fractures in the area of the meningeal artery. Zygomatic arch fractures are more visible on Towne's view. Characteristic swelling and lateral orbital bruising are typically present. Mandible fractures may be associated with dental misalignment or bleeding.

Which one of the following is a major advantage of second-generation (atypical) antipsychotics compared with first-generation antipsychotics? (check one) A. Less tardive dyskinesia B. Less monitoring for major side effects C. The availability of depot (intramuscular) formulations D. Lower cost E. Simpler dosing schedules

A recent expert consensus panel endorsed the use of second-generation antipsychotics rather than first-generation drugs. Tardive dyskinesia is much less common with the use of second-generation antipsychotics. Several of the second-generation drugs require monitoring for major side effects, however. For example, clozapine, shown by studies to be the most efficacious of the new class, causes granulocytopenia or agranulocytosis, requiring weekly and later biweekly monitoring of blood counts. Both classes have depot formulations for intramuscular administration every 2-4 weeks. Oral dosing of drugs from both classes varies from 1 to 3 times daily. First-generation antipsychotics cost less than second-generation drugs.

A 59-year-old male reports decreases in sexual desire and spontaneous erections, as well as reduced beard growth. The most appropriate test to screen for late-onset male hypogonadism is:

A serum total testosterone level is recommended as the initial screening test for late-onset male hypogonadism. Due to its high cost, a free testosterone level is recommended only if the total testosterone level is borderline and abnormalities in sex hormone-binding globulin are suspected. Follow-up LH and FSH levels help to distinguish primary from secondary hypogonadism.

Aortic Dissection Tx: Ascending vs. Descending

ASCENDING - Surgery DESCENDING - Beta Blocker

A 28-year-old white female consults you with a complaint of irregular heavy menstrual periods. A general physical examination, pelvic examination, and Papanicolaou test are normal and a pregnancy test is negative. A CBC and chemistry profile are also normal. The next step in her workup should be: (check one) A. endometrial aspiration B. dilatation and curettage C. LH and FSH assays D. administration of estrogen E. cyclic administration of progesterone for 3 months

Abnormal uterine bleeding is a relatively common disorder that may be due to functional disorders of the hypothalamus, pituitary, or ovary, as well as uterine lesions. However, the patient who is younger than 30 years of age will rarely be found to have a structural uterine defect. Once pregnancy, hematologic disease, and renal impairment are excluded, administration of intramuscular or oral progesterone will usually produce definitive flow and control the bleeding. No further evaluation should be necessary unless the bleeding recurs. Endometrial aspiration, dilatation and curettage, and other diagnostic procedures are appropriate for recurrent problems or for older women. Estrogen would only increase the problem, which is usually due to anovulation with prolonged estrogen secretion, producing a hypertrophic endometrium.

A 28-year-old male presents with the recent onset of intermittent urethral discharge accompanied by dysuria. He is heterosexual, has no prior history of a sexually transmitted infection, and acquired a new sexual partner a month ago. He has no regional lymphadenopathy or ulcers, and gentle milking of the urethra produces no discharge. Evaluation of a first-void urine specimen, however, reveals 15 WBCs/hpf. You treat him with oral azithromycin (Zithromax), 1 g in a single dose, and ceftriaxone (Rocephin), 125 mg intramuscularly. Test results for gonorrhea, Chlamydia, syphilis, HIV, and hepatitis B are negative. He returns 2 months later because his urethral discharge has persisted. He reports no relationships with a different sexual partner, and is confident that his current partner has only had sexual contact with him. You repeat the previous tests and again treat him with oral azithromycin. According to CDC testing and treatment guidelines, which one of the following drugs should be added to his treatment regimen? (check one) A. Metronidazole (Flagyl) B. Amoxicillin/clavulanate (Augmentin) C. Ciprofloxacin (Cipro) D. Trimethoprim/sulfamethoxazole (Bactrim, Septra) E. Cefixime (Suprax)

According to CDC guidelines, the initial workup for urethritis in men includes gonorrhea and Chlamydia testing of the penile discharge or urine, urinalysis with microscopy if no discharge is present, VDRL or RPR testing for syphilis, and HIV and hepatitis B testing. Empiric treatment for men with a purulent urethral discharge or a positive urine test (positive leukocyte esterase or ≥10 WBCs/hpf in the first-void urine sediment) includes azithromycin, 1 g orally as a single dose, OR doxycycline, 100 mg orally twice a day for 7 days, PLUS ceftriaxone, 125 mg intramuscularly, OR cefixime, 400 mg orally as a single dose. If the patient presents with the same complaint within 3 months, and does not have a new sexual partner, the tests obtained at his first visit should be repeated, and consideration should be given to obtaining cultures for Mycoplasma or Ureaplasma and Trichomonas from the urethra or urine. Treatment should include azithromycin, 500 mg orally once daily for 5 days, or doxycycline, 100 mg orally twice daily for 7 days, plus metronidazole, 2 g orally as a single dose.

Which one of the following is considered a contraindication to the use of beta-blockers for congestive heart failure? (check one) A. Mild asthma B. Symptomatic heart block C. New York Heart Association (NYHA) Class III heart failure D. NYHA Class I heart failure in a patient with a history of a previous myocardial infarction E. An ejection fraction <30%

According to several randomized, controlled trials, mortality rates are improved in patients with heart failure who receive beta-blockers in addition to diuretics, ACE inhibitors, and occasionally, digoxin. Contraindications to beta-blocker use include hemodynamic instability, heart block, bradycardia, and severe asthma. Beta-blockers may be tried in patients with mild asthma or COPD as long as they are monitored for potential exacerbations. Beta-blocker use has been shown to be effective in patients with NYHA Class II or III heart failure. There is no absolute threshold ejection fraction. Beta-blockers have also been shown to decrease mortality in patients with a previous history of myocardial infarction, regardless of their NYHA classification

A 70-year-old male with a history of hypertension and type 2 diabetes mellitus presents with a 2-month history of increasing paroxysmal nocturnal dyspnea and shortness of breath with minimal exertion. An echocardiogram shows an ejection fraction of 25%. Which one of the patients current medications should be discontinued? (check one) A. Lisinopril (Zestril) B. Pioglitazone (Actos) C. Glipizide (Glucotrol) D. Metoprolol (Toprol-XL) E. Repaglinide (Prandin)

According to the American Diabetes Association guidelines, thiazolidinediones (TZDs) are associated with fluid retention, and their use can be complicated by the development of heart failure. Caution is necessary when prescribing TZDs in patients with known heart failure or other heart diseases, those with preexisting edema, and those on concurrent insulin therapy (SOR C). Older patients can be treated with the same drug regimens as younger patients, but special care is required when prescribing and monitoring drug therapy. Metformin is often contraindicated because of renal insufficiency or heart failure. Sulfonylureas and other insulin secretagogues can cause hypoglycemia. Insulin can also cause hypoglycemia, and injecting it requires good visual and motor skills and cognitive ability on the part of the patient or a caregiver. TZDs should not be used in patients with New York Heart Association class III or IV heart failure.

A 74-year-old African-American female has moderately severe pain due to osteoarthritis. However, she is also on medication for a seizure disorder. When choosing medications to manage her chronic pain, which one of the following should be used with caution because of her history of seizures? (check one) A. Salsalate (Disalcid) B. Celecoxib (Celebrex) C. Hydrocodone (Lortab) D. Oxycodone (OxyContin) E. Tramadol (Ultram)

According to the American Geriatrics Society 2002 clinical practice guidelines for management of persistent pain in older persons, tramadol has efficacy and safety similar to those of equianalgesic doses of codeine and hydrocodone. However, because of the threat of seizures (rare but potential), tramadol should be used with caution in patients with a history of seizure disorder or those taking other medications that lower seizure thresholds.

A 52-year-old white male is being considered for pharmacologic treatment of hyperlipidemia because of an LDL cholesterol level of 180 mg/dL. Before beginning medication for his hyperlipidemia, he should be screened for: (check one) A. Hyperthyroidism B. Hypothyroidism C. Addison's disease D. Cushing's disease E. Pernicious anemia

According to the Summary of the National Cholesterol Education Program (NCEP) Adult Treatment Panel III Report of 2001, any person with elevated LDL cholesterol or any other form of hyperlipidemia should undergo clinical or laboratory assessment to rule out secondary dyslipidemia before initiation of lipid-lowering therapy. Causes of secondary dyslipidemia include diabetes mellitus, hypothyroidism, obstructive liver disease, chronic renal failure, and some medications.

A 72-year-old male with COPD presents to the emergency department with an acute exacerbation marked by increased sputum production and shortness of breath. His oxygen saturation is 88% on room air and he has diffuse inspiratory and expiratory wheezes bilaterally. In addition to oxygen and bronchodilators, which one of the following is most appropriate for this patient? (check one) A. No additional treatments B. Systemic corticosteroids only C. Inhaled corticosteroids only D. Systemic corticosteroids and antibiotics E. Inhaled corticosteroids and antibiotics

Acute exacerbations of COPD are very common, with most caused by superimposed infections. Supplemental oxygen, antibiotics, and bronchodilators are used for management. Systemic corticosteroids, either oral or parenteral, have been shown to significantly reduce treatment failures and improve lung function and dyspnea over the first 72 hours, although there is an increased risk of adverse drug reactions.

You are writing a prescription for amoxicillin for a 6-year-old female with acute otitis media. Her mother has had an anaphylactic reaction to penicillin in the past and is concerned that she may have passed this trait down to her daughter. You reassure her that this is not usually the case but warn her about potential signs of an allergic reaction. Which one of the following is the most concerning early symptom of a dangerous drug reaction? A) Tachycardia and elevated blood pressure B) Small, bright, erythematous macules diffusely over the trunk C) Pruritus around the mouth and on the palms of the hands and soles of the feet D) Eczematous patches in the antecubital and popliteal fossae E) Diarrhea with blood on the tissue paper (check one) A. Tachycardia and elevated blood pressure B. Small, bright, erythematous macules diffusely over the trunk C. Pruritus around the mouth and on the palms of the hands and soles of the feet D. Eczematous patches in the antecubital and popliteal fossae E) Diarrhea with blood on the tissue paper

Allergic reactions to medications have four primary mechanisms, referred to as Gell and Coombs classifications. The most frequent forms are type I reactions, which are immediate and mediated through IgE, and type IV reactions, which are delayed and mediated through T-cell hypersensitization. Severe type I reactions are often referred to as anaphylaxis and are the most likely to be life threatening with very little warning. Recognition of the early signs of anaphylaxis is the first step in preventing such catastrophes. Anaphylactic reactions result from a massive release of histamine and start with pruritus around the mouth, on the scalp, and on the palms and soles; flushing of the face and neck, with rhinitis and conjunctivitis; angioedema of the oral mucosa, especially of the pharynx and larynx; severe urticaria; dyspnea and bronchospasm (especially in known asthmatics); and hypotension. A delay in lifesaving therapy during this phase will result in full shock, hypotension, and death. Type IV reactions usually result in benign, diffuse erythematous macules on the trunk and proximal extremities, often referred to as a drug rash. These reactions infrequently become more severe and rarely are life threatening. In severe cases the lesions become painful and palpable, and may involve blistering, mucositis, and ecchymosis.

A 60-year-old male is recovering from a non-Q-wave myocardial infarction. He has a 40-pack-year smoking history, currently smokes a pack of cigarettes per day, and has a strong family history of coronary artery disease. Studies ordered by the cardiologist showed no indication for any coronary artery procedures. His BMI is 27.5 kg/m 2 and his blood pressure is 130/70 mm Hg. Laboratory tests reveal a fasting blood glucose level of 85 mg/dL, a total cholesterol level of 195 mg/dL, and an LDL-cholesterol level of 95 mg/dL. Which one of the following secondary prevention measures would be LEAST likely to improve this patient's cardiovascular outcome? (check one) A. A weight reduction diet B. A β-blocker C. A statin D. An antiplatelet agent E. Smoking cessation

Although dietary management may be appropriate, a weight reduction diet is not likely to improve this patient's cardiovascular outcome. In fact, even if this person were obese, there is insufficient evidence that weight reduction would decrease his cardiovascular mortality (SOR C). There is good evidence that the other options, even β-blockers in a patient with normal blood pressure, are indicated. All of these measures have evidence to support their usefulness for secondary prevention of coronary artery disease (SOR A).

A 62-year-old male has been taking omeprazole (Prilosec) for over a year for gastroesophageal reflux disease. He is asymptomatic and has had no problems tolerating the drug, but asks you about potential side effects, as well as the benefits of continuing therapy. It would be most accurate to tell him that omeprazole therapy is associated with which one of the following? (check one) A. A decreased rate of hip fracture B. Decreased vitamin B12 absorption C. A reduced likelihood of pneumonia D. A reduced likelihood of Clostridium difficile colitis E. An increased likelihood of iron deficiency anemia

Although proton pump inhibitors are the most effective treatment for patients with asymptomatic gastroesophageal reflux disease, there are several potential problems with prolonged therapy. Omeprazole is associated with an increased risk of community-acquired pneumonia and Clostridium difficile colitis. Omeprazole has also been shown to acutely decrease the absorption of vitamin B 12 , and it decreases calcium absorption, leading to an increased risk of hip fracture. The risk for Clostridium difficile colitis is also increased.

A 54-year-old female takes levothyroxine (Synthroid), 0.125 mg/day, for central hypothyroidism secondary to a pituitary adenoma. The nurse practitioner in your office orders a TSH level, which is found to be 0.1 mIU/mL (N 0.5-5.0). Which one of the following would you recommend? (check one) A. Decrease the dosage of levothyroxine B. Increase the dosage of levothyroxine C. Order a free T4 level D. Order a TRH stimulation test E. Repeat the TSH level in 3 months

Although uncommon, pituitary disease can cause secondary hypothyroidism. The characteristic laboratory findings are a low serum free T4 and a low TSH. A free T4 level is needed to evaluate the proper dosage of replacement therapy in secondary hypothyroidism. The TSH level is not useful for determining the adequacy of thyroid replacement in secondary hypothyroidism since the pituitary is malfunctioning. In the initial evaluation of secondary hypothyroidism, a TRH stimulation test would be useful if TSH failed to rise in response to stimulation. It is not necessary in this case, since the diagnosis has already been made.

Amiodarone (Cordarone) is most useful for which one of the following? (check one) A. Prophylactic perioperative use for emergency surgery B. Primary prevention of nonischemic cardiomyopathy C. Treatment of atrial flutter D. Treatment of multi-focal premature ventricular contractions following acute myocardial infarction E. Treatment of sustained ventricular tachyarrhythmias in patients with poor hemodynamic stability

Amiodarone is one of the most frequently prescribed antiarrhythmic medications in the U.S. It is useful in the acute management of sustained ventricular tachyarrhythmias, regardless of hemodynamic stability. Amiodarone is appropriate first-line treatment for atrial fibrillation only in symptomatic patients with left ventricular dysfunction and heart failure. It has a very limited role in the treatment of atrial flutter. The only role for prophylactic amiodarone is in the perioperative period of cardiac surgery. The use of prophylactic antiarrhythmic agents in the face of "warning dysrhythmias" or after myocardial infarction is no longer recommended. Prophylactic amiodarone is not indicated for primary prevention in patients with nonischemic cardiomyopathy.

Otits Media Tx

Amoxicillin is first line Amoxicillin + Clav

Treatment with which one of the following antihypertensive medications may mimic the effects of primary hyperparathyroidism? (check one) A. Amlodipine (Norvasc) B. Doxazosin (Cardura) C. Hydrochlorothiazide D. Lisinopril (Prinivil, Zestril) E. Metoprolol (Lopressor, Toprol-XL)

An elevated level of parathyroid hormone (or a level that is in an unexpected "normal" range) in a patient with an elevated calcium level generally indicates a diagnosis of primary hyperparathyroidism. However, these laboratory findings may also occur with lithium or thiazide use, tertiary hyperparathyroidism associated with end-stage renal failure, or familial hypocalciuric hypercalcemia, and a medical and family history should be obtained to assess these possibilities. The other medications listed do not cause hypercalcemia.

Which one of the following, when confirmed with a repeat test, meets the diagnostic criteria for diabetes mellitus? (check one) A. A fasting blood glucose level of 120 mg/dL B. A 2-hour value of 180 mg/dL on an oral glucose tolerance test C. A random glucose level of 180 mg/dL in a patient with symptoms of diabetes mellitus D. A positive urine dipstick for glucose E. A hemoglobin A1c of 7.0%

An international expert committee issued a report in 2009 recommending that a hemoglobin A1c level ≥6.5% be used to diagnose diabetes mellitus. Other criteria include a fasting plasma glucose level ≥126 mg/dL, a random glucose leve l≥200 mg/dL in a patient with symptoms of diabetes, or a 2-hour oral glucose tolerance test value ≥200 mg/dL. While a urine dipstick may be used to screen for diabetes, it is not a diagnostic test.

Which one of the following is an appropriate rationale for antibiotic treatment of Bordetella pertussis infections? (check one) A. It delays progression from the catarrhal stage to the paroxysmal stage B. It reduces the severity of symptoms C. It reduces the duration of illness D. It reduces the risk of transmission to others E. It reduces the need for hospitalization

Antibiotic treatment for pertussis is effective for eradicating bacterial infection but not for reducing the duration or severity of the disease. The eradication of infection is important for disease control because it reduces infectivity. Antibiotic treatment is thought to be most effective if started early in the course of the illness, characterized as the catarrhal phase. The paroxysmal stage follows the catarrhal phase. The CDC recommends macrolides for primary treatment of pertussis. The preferred antimicrobial regimen is azithromycin for 3-5 days or clarithromycin for 7 days. These regimens are as effective as longer therapy with erythromycin and have fewer side effects. Children under 1 month of age should be treated with azithromycin. There is an association between erythromycin and hypertrophic pyloric stenosis in young infants. Trimethoprim/sulfamethoxazole can be used in patients who are unable to take macrolides or where macrolide resistance may be an issue, but should not be used in children under the age of 2 months. Fluoroquinolones have been shown to reduce pertussis in vitro but have not been shown to be clinically effective (SOR A).

Which one of the following anticonvulsant medications is preferred for the treatment of mania or hypomania in patients with bipolar disorder? (check one) A. Phenytoin (Dilantin) B. Phenobarbital C. Valproic acid (Depakene) D. Gabapentin (Neurontin) E. Clonazepam (Klonopin)

Anticonvulsant medications are used in the treatment of various psychiatric disorders. Valproic acid is FDA-approved for the treatment of manic episodes associated with bipolar disorder. It has been shown in controlled studies to be significantly more effective than placebo. The initial dosage is 750 mg daily given in divided doses, and most individuals require between 1000 and 2500 mg daily. Carbamazepine has also been used to treat mania and is an alternative for individuals who cannot tolerate lithium or valproic acid. Clonazepam is used in the treatment of panic attacks, and gabapentin is used to treat anxiety. Both phenytoin and gabapentin are also used to treat peripheral neuropathy. The primary use of phenobarbital is as an anticonvulsant. Ref: Valproate and other anticonvulsants for psychiatric disorders.

A 24-year-old female has a history of mood swings over the past several months, which have created marital and financial problems, in addition to jeopardizing her career as a television news reporter. You have made a diagnosis of bipolar disorder, and she has finally accepted the need for treatment. However, she insists that you choose a drug that "won't make me fat."

Apirazole

A 24-year-old primigravida has nausea and vomiting associated with pregnancy. Which one of the following is recommended by the American Congress of Obstetricians and Gynecologists (ACOG) as first-line therapy? (check one) A. Droperidol (Inapsine) B. Ondansetron (Zofran) C. Prochlorperazine D. Metoclopramide (Reglan) E. Doxylamine (Unisom) and vitamin B6

Approximately 10% of women with nausea and vomiting during pregnancy require medication. Pharmacologic therapies that have been used include vitamin B6 , antihistamines, and prokinetic agents, as well as other medications. Randomized, placebo-controlled trials have shown that vitamin B6 is effective for this problem. The combination of vitamin B 6 and doxylamine was studied in more than 6000 patients and was associated with a 70% reduction in nausea and vomiting, with no evidence of teratogenicity. It is recommended by the American Congress of Obstetricians and Gynecologists as first-line therapy for nausea and vomiting in pregnancy. A combination pill was removed from the U.S. market in 1983 because of unjustified concerns about teratogenicity, but the medications can be bought separately over the counter. In rare cases, metoclopramide has been associated with tardive dyskinesia, and the FDA has issued a black-box warning concerning the use of this drug in general. The 5-HT3 -receptor antagonists, such as ondansetron, are being used for hyperemesis in pregnancy, but information is limited. Droperidol has been used for this problem in the past, but it is now used infrequently because of its risks, particularly heart arrhythmias.

While evaluating a stroke patient, you ask him to stick out his tongue. At first he is unable to do this, but a few moments later he performs this movement spontaneously. This defect is known as: (check one) A. apraxia B. agnosia C. expressive (Broca's) aphasia D. astereognosis

Apraxia is a transmission disturbance on the output side, which interferes with skilled movements. Even though the patient understands the request, he is unable to perform the task when asked, but may then perform it after a time delay. Agnosia is the inability to recognize previously familiar sensory input, and is a modality-bound deficit. For example, it results in a loss of ability to recognize objects. Aphasia is a language disorder, and expressive aphasia is a loss of the ability to express language. The ability to recognize objects by palpation in one hand but not the other is called astereognosis.

A 45-year-old male sees you for a routine annual visit and is found to have atrial fibrillation, with a ventricular rate of 70-75 beats/min. He is otherwise healthy, and a laboratory workup and echocardiogram are normal. Which one of the following would be the most appropriate management? (check one) A. Aspirin, 325 mg daily B. Warfarin (Coumadin), with a target INR of 2.0-3.0 C. Clopidogrel (Plavix), 75 mg daily D. Amiodarone (Cordarone), 200 mg daily E. Observation only

Atrial fibrillation is the most common arrhythmia, and its prevalence increases with age. The major risk with atrial fibrillation is stroke, and a patient's risk can be determined by the CHADS 2 score. CHADS stands for Congestive heart failure, Hypertension, Age >75, Diabetes mellitus, and previous Stroke or transient ischemic attack. Each of these is worth 1 point except for stroke, which is worth 2 points. A patient with 4 or more points is at high risk, and 2-3 points indicates moderate risk. Having ≤1 point indicates low risk, and this patient has 0 points. Low-risk patients should be treated with aspirin, 81-325 mg daily (SOR B). Moderate-or high-risk patients should be treated with warfarin. Amiodarone is used for rate control, and clopidogrel is used for vascular events not related to atrial fibrillatio

A 75-year-old African-American male with no previous history of cardiac problems complains of shortness of breath and a feeling of general weakness. His symptoms have developed over the past 24 hours. On physical examination you find a regular pulse with a rate of 160 beats/min. You note rales to the base of the scapula bilaterally, moderate jugular venous distention, and hepatojugular reflux. His blood pressure is 90/55 mm Hg; when he sits up he becomes weak and diaphoretic and complains of precordial pressure. An EKG reveals atrial flutter with 2:1 block. Management at this time should include: (check one) A. intravenous digoxin B. intravenous verapamil (Calan, Isoptin) C. amiodarone (Cordarone) D. electrical cardioversion E. insertion of a pacemaker

Atrial flutter is not ordinarily a serious arrhythmia, but this patient has heart failure manifested by rales, jugular venous distention, hepatojugular reflux, hypotension, and angina. Electrical cardioversion should be performed immediately. This is generally a very easy rhythm to convert. Digoxin and verapamil are appropriate in hemodynamically stable patients. A pacemaker for rapid atrial pacing may be beneficial if digitalis intoxication is the cause of atrial flutter, but this is unlikely in a patient with no previous history of cardiac problems. Amiodarone is not indicated in this clinical situation.

A 27-year-old white male has been in rehabilitation for C6 complete quadriplegia. His health had been good prior to a diving accident 2 months ago which caused his paralysis. The patient has been catheterized since admission and his recovery has been steady. His vital signs have been normal and stable. The nurse calls and tells you that for the past hour the patient has experienced sweating, rhinorrhea, and a pounding headache. His heart rate is 55/min and his blood pressure is 220/115 mm Hg. His temperature and respirations are reported as normal. There has been no vomiting and his neurologic examination is unchanged. The most likely diagnosis is: (check one) A. Cluster headache B. Autonomic hyperreflexia C. Sepsis D. Intracranial hemorrhage E. Progression of the spinal cord lesion

Autonomic hyperreflexia is characterized by the sudden onset of headache and hypertension in a patient with a lesion above the T6 level. There may be associated bradycardia, sweating, dilated pupils, blurred vision, nasal stuffiness, flushing, or piloerection. It usually occurs several months after the injury and has an incidence as high as 85% in quadriplegic patients. Frequently, it subsides within 3 years of injury, but it can recur at any time. Bowel and bladder distention are common causes. Hypertension is the major concern because of associated seizures and cerebral hemorrhage. Cluster headaches have a constant unilateral orbital localization. The pain is steady (non-throbbing) and lacrimation and rhinorrhea may be part of the syndrome. Sepsis is usually manifested by chills, fever, nausea, and vomiting. Common signs include tachycardia and hypotension rather than bradycardia and hypertension. Signs and symptoms of intracranial hemorrhage vary depending upon the site of the hemorrhage, but the unchanged neurologic status and the lack of a history of hypertension decrease the likelihood of this diagnosis. There are no neurologic findings or history which suggest progression of the patient's lesion at C6

An 8-year-old male presents with cervical lymphadenitis. He has a kitten at home and you are concerned about cat-scratch disease. Which one of the following antibiotics is most appropriate for treatment of Bartonella henselae infection? (check one) A. Azithromycin (Zithromax) B. Ceftriaxone (Rocephin) C. Amoxicillin/clavulanate (Augmentin) D. Doxycycline E. Clindamycin (Cleocin)

Azithromycin has been shown to reduce the duration of lymphadenopathy in cat-scratch disease (SOR B). Other antibiotics that have been used include rifampin, ciprofloxacin, trimethoprim/sulfamethoxazole, and gentamicin. Ceftriaxone, amoxicillin/clavulanate, doxycycline, and clindamycin are not effective in the treatment of Bartonella infection.

A 68-year-old African-American female with primary hypothyroidism is taking levothyroxine (Synthroid), 125 μg/day. Her TSH level is 0.2μU/mL (N 0.5-5.0). She has no symptoms of either hypothyroidism or hyperthyroidism. Which one of the following would be most appropriate at this point? (check one) A. Continuing levothyroxine at the same dosage B. Increasing the levothyroxine dosage C. Decreasing the levothyroxine dosage D. Discontinuing levothyroxine E. Ordering a free T 4

Because of the precise relationship between circulating thyroid hormone and pituitary TSH secretion, measurement of serum TSH is essential in the management of patients receiving levothyroxine therapy. Immunoassays can reliably distinguish between normal and suppressed concentrations of TSH. In a patient receiving levothyroxine, a low TSH level usually indicates overreplacement. If this occurs, the dosage should be reduced slightly and the TSH level repeated in 2-3 months' time. There is no need to discontinue therapy in this situation, and repeating the TSH level in 2 weeks would not be helpful. A free T4 level would also be unnecessary, since it is not as sensitive as a TSH level for detecting mild states of excess thyroid hormone.

Which one of the following therapeutic agents is most appropriate for daily use in the prevention of migraine headache? (check one) A. Dihydroergotamine (D.H.E. 45) B. Amitriptyline (Elavil) C. Sumatriptan (Imitrex) D. Aspirin/caffeine/butalbital (Fiorinal) E. Acetaminophen/hydrocodone bitartrate (Vicodin)

Beta-adrenergic blockers, antidepressants, anticonvulsants, calcium channel blockers, NSAIDs, and serotonin antagonists are the major classes of drugs used for preventive migraine therapy. All of these medications result in about a 50% reduction in the frequency of headaches. The other drugs listed are useful for the treatment of acute migraine, but not for prevention.

Which one of the following has been shown to decrease mortality late after a myocardial infarction? (check one) A. Nitrates B. Beta-blockers C. Digoxin D. Thiazide diuretics E. Calcium channel antagonists

Beta-blockers and ACE inhibitors have been found to decrease mortality late after myocardial infarction. Aspirin has been shown to decrease nonfatal myocardial infarction, nonfatal stroke, and vascular events. Nitrates, digoxin, thiazide diuretics, and calcium channel antagonists have not been found to reduce mortality after myocardial infarction.

Which one of the following classes of diabetes medications acts primarily by stimulating pancreatic insulin secretion? (check one) A. Biguanides, such as metformin (Glucophage) B. Thiazolidinediones, such as pioglitazone (Actos) C. DPP-4 inhibitors, such as sitagliptin (Januvia) D. Sulfonylureas, such as glipizide (Glucotrol) E. Amylin analogs, such as pramlintide (Symlin)

Biguanides and thiazolidinediones are insulin sensitizers that decrease hepatic glucose production and increase insulin sensitivity. Sulfonylureas and meglitinides stimulate pancreatic insulin secretion, while DPP-4 inhibitors prevent GLP-1 breakdown and slow the breakdown of some sugars. GLP-1 mimetics stimulate insulin secretion, suppress glucagon secretion, and promote β-cell production. Amylin analogs act with insulin to delay gastric emptying and they also inhibit glucagon release.

A 63-year-old white male sees you for an initial visit and is accompanied by his daughter, who is a patient of yours and scheduled the visit. The father recently relocated to be near the daughter after his wife died. He has well-controlled type 2 diabetes mellitus, but is otherwise healthy. Referring to the copy of the medical records they brought with them, the daughter notes that her father has received influenza vaccine in 3 of the past 5 years, but she can find no documentation that he ever had "the pneumonia vaccine." She asks if he should receive it at this visit. You advise them that he should receive pneumococcal vaccine: (check one) A. annually, along with influenza vaccine B. now and a repeat dose every 5 years C. every 5 years starting at age 65 D. now and a repeat dose once at age 68 E. only once, at age 65

Both the CDC and the American Academy of Family Physicians recommend that all adults over the age of 65 receive a single dose of pneumococcal polysaccharide vaccine. Immunization before the age of 65 is recommended for certain subgroups of adults, including institutionalized individuals over the age of 50; those with chronic cardiac or pulmonary disease, diabetes mellitus, anatomic asplenia, chronic liver disease, or kidney failure; and health-care workers. It is recommended that those receiving the vaccine before the age of 65 receive an additional dose at age 65 or 5 years after the first dose, whichever is later.

A 6-month-old white male is brought to your office because he has "blisters" in his diaper area. On examination, you find large bullae filled with cloudy yellow fluid. Some of the blisters have ruptured and the bases are covered with a thin crust. Which one of the following is most appropriate in the management of this condition? (check one) A. Rinsing diapers with a vinegar solution B. A topical antifungal agent C. Penicillin D. Trimethoprim/sulfamethoxazole (Bactrim, Septra)

Bullous impetigo is a localized skin infection characterized by large bullae; it is caused by phage group II Staphylococcus aureus. Cultures of fluid from an intact blister will reveal the causative agent. The lesions are caused by exfolatin, a local toxin produced by the S. aureus, and develop on intact skin. Complications are rare, but cellulitis occurs in <10% of cases. Strains of Staphylococcus associated with impetigo in the U.S. have little or no nephritogenic potential. Systemic therapy should be used in patients with widespread lesions. With the emergence of MRSA, trimethoprim/sulfamethoxazole and clindamycin are options for outpatient therapy. Intravenous vancomycin can be used to treat hospitalized patients with more severe infections.

A 74-year-old male presents with a 4-day history of diarrhea that he had initially thought was "a 24-hour virus." He states that the onset of his illness included nausea, one episode of vomiting, and profuse diarrhea. He has felt feverish and has been having abdominal cramps. He does not recall eating anything unusual and has not traveled recently. On examination he appears uncomfortable, but in no real distress. His oral temperature is 37.1°C (98.8°F), blood pressure 134/82 mm Hg, and pulse rate 100 beats/min. He has lost 4 kg (9 lb) since his last visit 2 months earlier. His abdomen is soft, with hyperactive bowel sounds and mild diffuse tenderness on palpation. A CBC and basic metabolic profile are normal. Which one of the following is the most likely cause of this patient's illness? (check one) A. Norwalk-like virus (Norovirus) B. Shigella C. Campylobacter D. Escherichia coli O157:H7 E. Staphyloccocus aureus

Campylobacter jejuni is one of the most common causes of bacterial foodborne illnesses, estimated to affect 1 million Americans annually. Undercooked or improperly handled chicken is most often implicated as the source; surveys have demonstrated that between 20% and 100% of all retail chicken sold in the United States is contaminated. The infection is generally isolated and sporadic, occurs more frequently at the extremes of age, is most common during the summer months, and affects males disproportionately. Symptoms typically begin 2-5 days following exposure. Diarrhea is the predominant symptom, with a lesser degree of nausea and vomiting. Up to 10 days is required for full recovery. While Escherichia coli O157:H7 and Shigella may cause a similar illness, both generally present with bloody diarrhea. E. coli O157:H7 is most often transmitted in contaminated undercooked beef, and Shigella is usually spread in a fecal-oral pattern or via contaminated water. The peripheral WBC count is typically increased substantially in shigellosis. Staphylococcus aureus produces an enterotoxin in food that causes the onset of nausea, vomiting, and diarrhea within hours of ingestion and clears within 24-48 hours. Norovirus is a very common cause of acute viral gastroenteritis, usually with more vomiting than diarrhea. It spreads person to person, and patients usually recover within 24 hours.

A middle-aged hairdresser presents with a complaint of soreness of the proximal nail folds of several fingers on either hand, which has slowly worsened over the last 6 months. The nails appear thickened and distorted. Otherwise she is healthy and has no evidence of systemic disease. Which one of the following would be the most effective initial treatment? (check one) A. Soaking in a dilute iodine solution twice daily to cleanse and sterilize the nail beds B. Oral amoxicillin/clavulanate (Augmentin) for up to 4-6 weeks C. Topical betamethasone dipropionate (Diprolene) applied twice daily to the nail folds for 3-4 weeks D. Evaluation for HIV, hepatitis C, psoriasis, and rheumatoid arthritis

Chronic paronychia is a common condition in workers whose hands are exposed to chemical irritants or are wet for long periods of time. This patient is an otherwise healthy hairdresser, with frequent exposure to irritants. The patient should be advised to avoid exposure to harsh chemicals and water. In addition, the use of strong topical corticosteroids over several weeks can greatly reduce the inflammation, allowing the nail folds to return to normal and helping the cuticles recover their natural barrier to infection. Soaking in iodine solution would kill bacteria, but would also perpetuate the chronic irritation. Because the condition is related to chemical and water irritation, a prolonged course of antibiotics should not be the first treatment step, and could have serious side effects. There is no need to explore less likely autoimmune causes for nail changes at this time.

A 40-year-old female with chronic plaque psoriasis requests topical treatment. Which one of the following topical therapies would be most effective and have the fewest adverse effects? (check one) A. High-potency corticosteroids B. Tazarotene (Tazorac) C. Coal tar polytherapy D. Anthralin

Chronic plaque psoriasis is the most common type of psoriasis and is characterized by redness, thickness, and scaling. A variety of treatments were found to be more effective than placebo, but the best results were produced by topical vitamin D analogues and topical corticosteroids. Vitamin D and high-potency corticosteroids were equally effective when compared head to head, but the corticosteroids produced fewer local reactions (SOR A).

A 70-year-old male presents to your office for a follow-up visit for hypertension. He was started on lisinopril (Prinivil, Zestril), 20 mg daily, 1 month ago. Laboratory tests from his last visit, including a CBC and a complete metabolic panel, were normal except for a serum creatinine level of 1.5 mg/dL (N 0.6-1.5). A follow-up renal panel obtained yesterday shows a creatinine level of 3.2 mg/dL and a BUN of 34 mg/dL (N 8-25). Which one of the following is the most likely cause of this patient's increased creatinine level? (check one) A. Bilateral renal artery stenosis B. Coarctation of the aorta C. Essential hypertension D. Hyperaldosteronism E. Pheochromocytoma

Classic clinical clues that suggest a diagnosis of renal-artery stenosis include the onset of stage 2 hypertension (blood pressure >160/100 mm Hg) after 50 years of age or in the absence of a family history of hypertension; hypertension associated with renal insufficiency, especially if renal function worsens after the administration of an agent that blocks the renin-angiotensin-aldosterone system; hypertension with repeated hospital admissions for heart failure; and drug-resistant hypertension (defined as blood pressure above the goal despite treatment with three drugs of different classes at optimal doses). The other conditions mentioned do not cause a significant rise in serum creatinine after treatment with an ACE inhibitor.

A healthy 48-year-old bookkeeper who works in a medical office has a positive PPD on routine yearly screening. Which one of the following would be most appropriate at this point? (check one) A. A chest radiograph B. A repeat PPD C. Treatment with isoniazid and one other antituberculous drug for 12 months D. Anergy testing

Clinical evaluation and a chest radiograph are recommended in asymptomatic patients with a positive PPD (SOR C). A two-step PPD is performed on those at high risk whose initial test is negative. Asymptomatic patients with a positive PPD and an abnormal chest film should have a sputum culture for TB, but a culture is not required if the chest film is negative. Persons with a PPD conversion should be encouraged to take INH for 9 months with proper medical supervision. Patients with a negative PPD who are still at high risk for TB, especially HIV-positive patients, could be evaluated for anergy, but it is not recommended at this time.

Which one of the following is most predictive of increased perioperative cardiovascular events associated with noncardiac surgery in the elderly? (check one) A. An age of 80 years B. Left bundle-branch block C. Atrial fibrillation with a rate of 80 beats/min D. A history of previous stroke E. Renal insufficiency (creatinine 2.0 mg/dL)

Clinical predictors of increased perioperative cardiovascular risk for elderly patients include major risk factors such as unstable coronary syndrome (acute or recent myocardial infarction, unstable angina), decompensated congestive heart failure, significant arrhythmia (high-grade AV block, symptomatic ventricular arrhythmia, supraventricular arrhythmias with uncontrolled ventricular rate), and severe valvular disease. Intermediate predictors are mild angina, previous myocardial infarction, compensated congestive heart failure, diabetes mellitus, and renal insufficiency. Minor predictors are advanced age, an abnormal EKG, left ventricular hypertrophy, left bundle-branch block, ST and T-wave abnormalities, rhythm other than sinus, low functional capacity, history of stroke, and uncontrolled hypertension. Ref: Schroeder BM: Updated guidelines for perioperative cardiovascular evaluation for noncardiac surgery

A 75-year-old male develops a mild Clostridium difficile infection and is treated with 10 days of metronidazole (Flagyl), 500 mg orally 3 times daily. The diarrhea recurs 10 days after he completes the course of treatment. Which one of the following would be most appropriate? (check one) A. Repeat the course of metronidazole B. Repeat the course of metronidazole and add vancomycin C. Administer vancomycin intravenously D. Prescribe loperamide (Imodium), 4 mg twice daily as needed E. Prescribe a probiotic

Clostridium difficile infection is more common with aging and can be treated with either metronidazole or vancomycin daily. For mild recurrent disease, repeating the course of the original agent is appropriate (SOR B). Multiple recurrences or severe disease warrants the use of both agents. The effectiveness of probiotics such as Lactobacillus remains uncertain. Intravenous vancomycin has not been effective. Antiperistaltic drugs should be avoided.

Which one of the following antipsychotic medications is most likely to cause agranulocytosis? (check one) A. Clozapine (Clozaril) B. Aripiprazole (Abilify) C. Risperidone (Risperdal) D. Olanzapine (Zyprexa)

Clozapine was the first atypical antipsychotic drug, so designated because it has antipsychotic effects without the adverse effects on movement seen with first-generation agents, in addition to having enhanced therapeutic efficacy compared with first-generation drugs. Because of these advantages, it was introduced into clinical practice in the United States despite a serious known adverse effect: an increased incidence of agranulocytosis. Although only clozapine causes agranulocytosis in a substantial proportion of patients, many second-generation drugs produce clinically significant weight gain.

A 69-year-old female sees you for an annual examination. She asks you to look at her toes, and you note a fungal infection in five toenails. She says the condition is painful and limits her ability to complete her morning walks. She asks for treatment that will allow her to resume her daily walks as soon as possible. Her only other medical problem is allergic rhinitis which is well controlled. (check one) A. Oral griseofulvin ultramicrosize (Gris-PEG) daily for 12 weeks B. Oral terbinafine (Lamisil) daily for 12 weeks C. Topical terbinafine (Lamisil AT) daily for 12 weeks D. Topical ciclopirox (Penlac Nail Lacquer) daily for 12 weeks E. Toenail removal

Continuous therapy with oral terbinafine for 12 weeks has the highest cure rate and best long-term resolution rate of the therapies listed. Other agents and pulsed dosing regimens have lower cure rates. Topical creams are not appropriate for onychomycosis because the infection resides in the cell of the toenail. Antifungal nail lacquers have a lower cure rate than systemic therapy and should be used only when oral agents would not be safe. Toenail removal is reserved for patients with an isolated infected nail or in cases involving a dermatophytoma.

You see a 32-year-old white female for her first visit. She presents with numerous complaints which do not conform to patterns seen in organic disease. She states that she has seen several physicians and describes a changing set of symptoms. Although she appears to be well, she claims to have been "sickly" for years. From her affect, you suspect that she is depressed. The most likely diagnosis is: (check one) A. Conversion reaction B. Chronic somatization disorder C. Schizophrenia with multiple somatic delusions D. Histrionic personality E. Primary hypochondriasis

Conversion disorder usually involves a single symptom which is neurologic or pain-related. Symptoms of chronic somatization differ from psychoses in that the symptoms of the psychotic patient are bizarre and more vivid, persist over time, are unaltered by reasoned argument, and are not congruent with the patient's social or cultural background. The delusional nature of psychotic somatic symptoms usually unfolds as the patient talks. The essential feature of the histrionic (hysterical) personality is a pervasive pattern of excessive emotionality and attention seeking. People with this disorder constantly seek to be the center of attention. Emotions are often expressed with inappropriate exaggeration. People with this disorder tend to be very self-centered and have little tolerance for delayed gratification. These people are typically attractive and seductive, often to the point of looking flamboyant and acting inappropriately. Features of primary hypochondriasis include the patient's fixed conviction that he or she is ill, the interpretation of all somatic changes as confirmation of this, and a relentless pursuit of medical assistance despite persistent dissatisfaction with the results. The patient's symptoms remain consistent for years. Physicians frequently feel overwhelmed when initially presented with a patient with somatization disorder. This disorder begins before age 30 and is rarely seen in males. The patient complains of multiple symptoms which involve many organ systems and do not readily conform to patterns seen in organic diseases. The patient skips back and forth from symptom to symptom during the interview. Anxiety and depressed mood are frequent in this disorder, and suicide attempts are common

A 68-year-old white male with diabetes mellitus is hospitalized after suffering a right middle cerebral artery stroke. A nurse in the intensive-care unit calls to advise you that his blood pressure is 200/110 mm Hg. You should: (check one) A. continue monitoring the patient B. administer labetalol (Trandate) C. administer nicardipine (Cardene) D. administer nitroprusside (Nitropress) E. administer nitroglycerin

Current American Heart Association guidelines for blood pressure control in stroke patients advise monitoring with no additional treatment for patients with a systolic blood pressure <220 mm Hg or a diastolic blood pressure <120 mm Hg. The elevated blood pressure is thought to be a protective mechanism that increases cerebral perfusion, and lowering the blood pressure may increase morbidity.

Which one of the following medications should be discontinued in a patient with diabetic gastroparesis? (check one) A. Exenatide (Byetta) B. Benazepril (Lotensin) C. Metformin (Glucophage) D. Hydrochlorothiazide E. Prochlorperazine maleate

Delayed gastric emptying may be caused or exacerbated by medications for diabetes, including amylin analogues (e.g., pramlintide) and glucagon-like peptide 1 (e.g., exenatide). Delayed gastric emptying has a direct effect on glucose metabolism, in addition to being a means of reducing the severity of postprandial hyperglycemia. In a clinical trial of exenatide, nausea occurred in 57% of patients and vomiting occurred in 19%, which led to the cessation of treatment in about one-third of patients. The other medications listed do not cause delayed gastric emptying.

Which one of the following tinea infections in children always requires systemic antifungal therapy? (check one) A. Tinea cruris B. Tinea corporis C. Tinea capitis D. Tinea pedis E. Tinea versicolor

Dermatophyte infections caused by aerobic fungi produce infections in many areas. Tinea capitis requires systemic therapy to penetrate the affected hair shafts. Tinea cruris and tinea pedis rarely require systemic therapy. Extensive outbreaks of tinea corporis and tinea versicolor benefit from both oral and topical treatment (SOR A), but more localized infections require only topical treatment

A patient who takes fluoxetine (Prozac), 40 mg twice daily, develops shivering, tremors, and diarrhea after taking an over-the-counter cough and cold medication. On examination he has dilated pupils and a heart rate of 110 beats/min. His temperature is normal. Which one of the following medications in combination with fluoxetine could contribute to this patient's symptoms? (check one) A. Dextromethorphan B. Pseudoephedrine C. Phenylephrine D. Guaifenesin E. Diphenhydramine (Benadryl)

Dextromethorphan is commonly found in cough and cold remedies, and is associated with serotonin syndrome. SSRIs such as fluoxetine are also associated with serotonin syndrome, and there are many other medications that increase the risk for serotonin syndrome when combined with SSRIs. The other medications listed here are not associated with serotonin syndrome, however.

For 2 weeks, a 62-year-old male with biopsy-documented cirrhosis and ascites has had diffuse abdominal discomfort, fever, and night sweats. His current medications are furosemide (Lasix) and spironolactone (Aldactone). On examination, his temperature is 38.0° C (100.4° F), blood pressure 100/60 mm Hg, heart rate 92 beats/min and regular. The heart and lung examination is normal. The abdomen is soft with vague tenderness in all quadrants. There is no rebound or guarding. The presence of ascites is easily verified. Bowel sounds are quiet. The rectal examination is normal, and the stool is negative for occult blood. You perform diagnostic paracentesis and send a sample of fluid for analysis. Which one of the following findings would best establish the suspected diagnosis of spontaneous bacterial peritonitis? (check one) A. pH <7.2 B. Bloody appearance C. Neutrophil count >300/mL D. Positive cytology E. Total protein >1 g/dL

Diagnostic paracentesis is recommended for patients with ascites of recent onset, as well as for those with chronic ascites who present with new clinical findings such as fever or abdominal pain. A neutrophil count >250/mL is diagnostic for peritonitis. Once peritonitis is diagnosed, antibiotic therapy should be started immediately without waiting for culture results. Bloody ascites with abnormal cytology may be seen with hepatoma, but is not typical of peritonitis. The ascitic fluid pH does not become abnormal until well after the neutrophil count has risen, so it is a less reliable finding for treatment purposes. A protein level >1 g/dL is actually evidence against spontaneous bacterial peritonitis.

An 85-year-old white male with terminal pancreatic cancer is expected to survive for another 2 weeks. His pain has been satisfactorily controlled with sustained-release morphine. He has now developed a disturbed self-image, hopelessness, and anhedonia, and has told family members that he has thought about suicide. Psychomotor retardation is also noted. His family is supportive. His daughter feels he is depressed, while his son feels this is more of a grieving process. Which one of the following would be most appropriate for managing this problem? (check one) A. Reassurance B. Alprazolam (Xanax) C. Trazodone (Desyrel) D. Olanzapine (Zyprexa) E. Methylphenidate (Ritalin)

Distinguishing between preparatory grief and depression in a dying patient is not always simple. Initially one should evaluate for unresolved physical symptoms and treat any that are present. If the patient remains in distress, mood should be evaluated. If it waxes and wanes with time and if self-esteem is normal, this is likely preparatory grief. The patient may have fleeting thoughts of suicide and likely will express worry about separation from loved ones. This usually responds to counseling. In patients with anhedonia, persistent dysphoria, disturbed self-image, hopelessness, poor sense of self-worth, rumination about death and suicide, or an active desire for early death, depression is the problem. For patients who are expected to live only a few days, psychostimulants such as methylphenidate should be used. For those who are expected to survive longer, SSRIs are a good choice.

Isolated Systolic HTN Tx

Diuretic

African American HTN Treatment

Diuretic or CCB (not Labetalol or ACE Inhibitor)

Asymptomatic Aortic Stenosis

Do nothing

Which one of the following is the most effective drug for the treatment of alcohol dependence? (check one) A. Disulfiram (Antabuse) B. Diazepam (Valium) C. Amitriptyline (Elavil) D. Fluoxetine (Prozac) E. Naltrexone (ReVia)

Drug therapy should be considered for all patients with alcohol dependence who do not have medical contraindications to the use of the drug and who are willing to take it. Of the several drugs studied for the treatment of dependence, the evidence of efficacy is strongest for naltrexone and acamprosate. Naltrexone is currently available in the U.S.; acamprosate and tiapride are currently available in Europe but not in the U.S. Ref: Swift RM: Drug therapy for alcohol dependence.

A 64-year-old African-American male presents with persistent pleuritic pain. The patient does not feel well in general and has had a low-grade fever of around 100°F (38°C). His medications include simvastatin (Zocor), lisinopril (Prinivil, Zestril), low-dose aspirin, spironolactone (Aldactone), furosemide (Lasix), isosorbide mononitrate (Imdur), hydralazine, carvedilol (Coreg), and nitroglycerin as needed. A chest radiograph is normal and does not demonstrate a pneumothorax. Further evaluation rules out pulmonary embolus, pneumonia, and myocardial infarction. A diagnosis of pleurisy is made. Which one of the patient's medications could be related to this condition? (check one) A. Hydralazine B. Simvastatin C. Lisinopril D. Spironolactone E. Carvedilol

Drug-induced pleuritis is one cause of pleurisy. Several drugs are associated with drug-induced pleural disease or drug-induced lupus pleuritis. Drugs that may cause lupus pleuritis include hydralazine, procainamide, and quinidine. Other drugs known to cause pleural disease include amiodarone, bleomycin, bromocriptine, cyclophosphamide, methotrexate, minoxidil, and mitomycin.

What is the most common cause of erythema multiforme, accounting for more than 50% of cases? (check one) A. Candida albicans B. Herpes simplex virus C. Mycoplasma pneumoniae D. Penicillin therapy E. Sulfonamide therapy

Erythema multiforme usually occurs in adults 20-40 years of age, although it can occur in patients of all ages. Herpes simplex virus (HSV) is the most commonly identified cause of this hypersensitivity reaction, accounting for more than 50% of cases.

Painful ingrown toenails that display granulation tissue and lateral nail fold hypertrophy are best treated by: (check one) A. Antibiotic therapy B. Cotton-wick elevation of the affected nail corner C. Removal of the entire nail D. Excision of the lateral nail plate combined with lateral matricectomy

Excision of the lateral nail plate with lateral matricectomy yields the best results in the treatment of painful ingrown toenails that display granulation tissue and lateral nail fold hypertrophy. Antibiotic therapy and cotton-wick elevation are acceptable for very mildly inflamed ingrown toenails. Partial nail avulsion often leaves a spicule of nail that will grow and become an ingrown nail. Phenol produces irregular tissue destruction and significant inflammation and discharge after the matricectomy procedure.

A 70-year-old male complains of lower-extremity pain. Increased pain with which one of the following would be most consistent with lumbar spinal stenosis? (check one) A. Lumbar spine extension B. Lumbar spine flexion C. Internal hip rotation D. Pressure against the lateral hip and trochanter E. Walking uphill

Extension that increases lumbar lordosis decreases the cross-sectional area of the spinal canal, thereby compressing the spinal cord further. Walking downhill can cause this. Spinal flexion that decreases lordosis has the opposite effect, and will usually improve the pain, as will sitting. Pain with internal hip rotation is characteristic of hip arthritis and is often felt in the groin. Pain in the lateral hip is more typical of trochanteric bursitis. Increased pain walking uphill is more typical of vascular claudication.

You are asked to perform a preoperative evaluation on a 55-year-old white female with type 2 diabetes mellitus prior to elective femoral-anterior tibial artery bypass surgery. She is unable to climb a flight of stairs or do heavy work around the house. She denies exertional chest pain, and is otherwise healthy. Based on current guidelines, which one of the following diagnostic studies would be appropriate prior to surgery because the results could alter the management of this patient? (check one) A. Pulmonary function studies B. Coronary angiography C. Carotid angiography D. A dipyridamole-thallium scan E. A hemoglobin A1c level

Family physicians are often asked to perform a preoperative evaluation prior to noncardiac surgery. This requires an assessment of the perioperative cardiovascular risk of the procedure involved, the functional status of the patient, and clinical factors that can increase the risk, such as diabetes mellitus, stroke, renal insufficiency, compensated or prior heart failure, mild angina, or previous myocardial infarction. This patient is not undergoing emergency surgery, nor does she have an active cardiac condition; however, she is undergoing a high-risk procedure (>5% risk of perioperative myocardial infarction) with vascular surgery. As she cannot climb a flight of stairs or do heavy housework, her functional status is <4 METs, and she should be considered for further evaluation. The patient's diabetes is an additional clinical risk factor. With vascular surgery being planned, appropriate recommendations include proceeding with the surgery with heart rate control, or performing noninvasive testing if it will change the management of the patient. Coronary angiography is indicated if the noninvasive testing is abnormal. Pulmonary function studies are most useful in patients with underlying lung disease or those undergoing pulmonary resection. Hemoglobin A1c is a measure of long-term diabetic control and is not particularly useful perioperatively. Carotid angiography is not indicated in asymptomatic patients being considered for lower-extremity vascular procedures.

Which one of the following is the most reliable clinical symptom of uterine rupture? (check one) A. Sudden, tearing uterine pain B. Vaginal bleeding C. Loss of uterine tone D. Fetal distress

Fetal distress has proven to be the most reliable clinical symptom of uterine rupture. The "classic" signs of uterine rupture such as sudden, tearing uterine pain, vaginal hemorrhage, and loss of uterine tone or cessation of uterine contractions are not reliable and are often absent. Pain and bleeding occur in as few as 10% of cases. Even ruptures monitored with an intrauterine pressure catheter fail to show loss of uterine tone. Signs of fetal distress are often the only manifestation of uterine rupture.

A 28-year-old female sees you with a complaint of irregular menses. She has not had a menstrual period for 6 months. She is also concerned about weight gain, worsening acne, and dark hair on her upper lip, chin, and periareolar region. She is also interested in becoming pregnant soon. The patient tells you she has started an exercise program, which has helped with weight loss, but she continues to have amenorrhea. She has a negative urine β-hCG test, a mild elevation in free testosterone levels, and glucose intolerance. Which one of the following would you consider initially for inducing ovulation? (check one) A. Insulin B. Metformin (Glucophage) C. Ethinyl estradiol/norgestimate (Ortho Tri-Cyclen) D. Glipizide (Glucotrol) E. Spironolactone (Aldactone)

First-line agents for ovulation induction and treatment of infertility in patients with polycystic ovary syndrome (PCOS) include metformin and clomiphene, alone or in combination, as well as rosiglitazone (SOR A). In one study of nonobese women with PCOS, metformin was found to be more effective than clomiphene for improving the rate of conception (level of evidence 1b). However, the treatment of infertile women with PCOS remains controversial. One recent group of experts recommended that metformin use for ovulation induction in PCOS be restricted to women with glucose intolerance (SOR C). Oral contraceptives are commonly used to treat menstrual irregularities in women with PCOS; however, there are few studies supporting their use, and they would not be appropriate for ovulation induction. Spironolactone is a first-line agent for treatment of hirsutism (SOR A) and has shown promise in treating menstrual irregularities, but is not commonly recommended for ovulation induction. There is a high prevalence of insulin resistance in women with PCOS, as measured by glucose intolerance; insulin-sensitizing agents are therefore indicated, but not insulin or sulfonylurea medications.

A 27-year-old white female has a 10-year history of significant premenstrual dysphoria. Her condition has significantly worsened in the past 3 years, to the point that it is endangering her marriage of 5 years. Her symptoms are worse for the 10 days prior to her menstrual period and are gone by day 2 of her period. She has tried several measures without success, including birth control pills, various herbal preparations, and counseling at a woman's health center. Which one of the following is most likely to improve her symptoms? (check one) A. Reduction of caffeine and refined sugar intake B. Alprazolam (Xanax) C. Bupropion (Wellbutrin) D. Progesterone for 2 weeks starting at about the time of ovulation E. Fluoxetine (Prozac, Serafem) for the last 2 weeks of the menstrual cycle

Fluoxetine

Which one of the following should be used first for ventricular fibrillation when an initial defibrillation attempt fails? (check one) A. Amiodarone (Cordarone) B. Lidocaine (Xylocaine) C. Adenosine (Adenocard) D. Vasopressin (Pitressin) E. Magnesium

For persistent ventricular fibrillation (VF), in addition to electrical defibrillation and CPR, patients should be given a vasopressor, which can be either epinephrine or vasopressin. Vasopressin may be substituted for the first or second dose of epinephrine. Amiodarone should be considered for treatment of VF unresponsive to shock delivery, CPR, and a vasopressor. Lidocaine is an alternative antiarrhythmic agent, but should be used only when amiodarone is not available. Magnesium may terminate or prevent torsades de pointes in patients who have a prolonged QT interval during normal sinus rhythm. Adenosine is used for the treatment of narrow complex, regular tachycardias and is not used in the treatment of ventricular fibrillation.

A 46-year-old female presents to your office with a 2-week history of pain in her left shoulder. She does not recall any injury, and the pain is present when she is resting and at night. Her only chronic medical problem is type 2 diabetes mellitus. On examination, she has limited movement of the shoulder and almost complete loss of external rotation. Radiographs of the shoulder are normal, as is her erythrocyte sedimentation rate. Which one of the following is the most likely diagnosis? (check one) A. Frozen shoulder B. Torn rotator cuff C. Impingement syndrome D. Chronic posterior shoulder dislocation E. Osteoarthritis

Frozen shoulder is an idiopathic condition that most commonly affects patients between the ages of 40 and 60. Diabetes mellitus is the most common risk factor for frozen shoulder. Symptoms include shoulder stiffness, loss of active and passive shoulder rotation, and severe pain, including night pain. Laboratory tests and plain films are normal; the diagnosis is clinical (SOR C). Frozen shoulder is differentiated from chronic posterior shoulder dislocation and osteoarthritis on the basis of radiologic findings. Both shoulder dislocation and osteoarthritis have characteristic plain film findings. A patient with a rotator cuff tear will have normal passive range of motion. Impingement syndrome does not affect passive range of motion, but there will be pain with elevation of the shoulder.

A previously healthy 82-year-old male is brought to your office by his daughter after a recent fall while getting up to go to the bathroom in the middle of the night. The patient denies any history of dizziness, chest pain, palpitations, or current injury. He has a history of bilateral dense cataracts. On examination, he is found to have an increased stance width and walks carefully and cautiously with his arms and legs abducted. A timed up-and-go test is performed, wherein the patient is asked to rise from a chair without using his arms, walk 3 meters, turn, return to his chair, and sit down. It takes the patient 25 seconds and he is noted to have an "en bloc" turn. Which one of the following is the most likely cause of this patient's gait and balance disorder? (check one) A. Visual impairment B. Cerebellar degeneration C. Frontal lobe degeneration D. Parkinson's disease E. Motor neuropathy

Gait and balance disorders are one of the most common causes of falls in older adults. Correctly identifying gait and balance disorders helps guide management and may prevent consequences such as injury, disability, loss of independence, or decreased quality of life. The "Timed Up and Go" test is a reliable diagnostic tool for gait and balance disorders and is quick to administer. A time of <10 seconds is considered normal, a time of >14 seconds is associated with an increased risk of falls, and a time of >20 seconds usually suggests severe gait impairment. This patient has the cautious gait associated with visual impairment. It is characterized by abducted arms and legs; slow, careful, "walking on ice" movements; a wide-based stance; and "en bloc" turns. Patients with cerebellar degeneration have an ataxic gait that is wide-based and staggering. Frontal lobe degeneration is associated with gait apraxia that is described as "magnetic," with start and turn hesitation and freezing. Parkinson's disease patients have a typical gait that is short-stepped and shuffling, with hips, knees, and spine flexed, and may also exhibit festination and "en bloc" turns. Motor neuropathy causes a "steppage" gait resulting from foot drop with excessive flexion of the hips and knees when walking, short strides, a slapping quality, and frequent tripping.

Which one of the following is recommended for the treatment of intravaginal genital warts in pregnant women? (check one) A. Imiquimod 5% cream (Aldara) B. Podofilox 0.5% solution (Condylox) C. Podophyllin 10%-25% in tincture of benzoin (Podofin) D. Cryotherapy with liquid nitrogen E. Interferon-alpha

Genital warts can proliferate and fragment during pregnancy, and many specialists recommend that they be eliminated. Imiquimod, podophyllin, and podofilox are not recommended for use during pregnancy. For the treatment of vaginal warts, the Centers for Disease Control and Prevention (CDC) recommends the use of cryotherapy. Liquid nitrogen, rather than a cryoprobe, should be used to avoid possible vaginal perforation and subsequent fistula formation. An alternative is the use of trichloroacetic acid or bichloroacetic acid carefully applied to the lesions to avoid damage to adjacent tissue. Interferon is no longer recommended for routine use in treating genital warts, due to a high frequency of systemic adverse effects.

A 45-year-old female presents with a 3-month history of hoarseness that is not improving. She works as a high-school teacher. The most appropriate management at this time would be: (check one) A. voice therapy B. azithromycin (Zithromax) C. a trial of inhaled corticosteroids D. a trial of a proton pump inhibitor E. laryngoscopy

Hoarseness most commonly affects teachers and older adults. The cause is usually benign, but extended symptoms or certain risk factors should prompt evaluation; specifically, laryngoscopy is recommended when hoarseness does not resolve within 3 months or when a serious underlying cause is suspected (SOR C). The American Academy of Otolaryngology/Head and Neck Surgery Foundation guidelines state that antireflux medications should not be prescribed for patients with hoarseness without reflux symptoms (SOR C). Antibiotics should not be used, as the condition is usually caused by acute laryngitis or an upper respiratory infection, and these are most likely to be viral. Inhaled corticosteroids are a common cause of hoarseness. Voice therapy should be reserved for patients who have undergone laryngoscopy first (SOR A).

Which one of the following drugs used to treat rheumatoid arthritis can delay the progression of the disease? (check one) A. Aspirin B. Ibuprofen C. Indomethacin (Indocin) D. Capsaicin (Zostrix) E. Hydroxychloroquine (Plaquenil)

Hydroxychloroquine, originally developed as an antimalarial drug, is a well-known disease-modifying agent that can slow the progression of rheumatoid arthritis. Aspirin, indomethacin, and ibuprofen are anti-inflammatory agents. They relieve pain and improve mobility, but do not alter the progression of the disease. Capsaicin, a topical substance-P depleter, can relieve pain symptoms.

Which one of the following medications has the best evidence for preventing hip fracture? (check one) A. Ibandronate (Boniva) B. Raloxifene (Evista) C. Denosumab (Prolia) D. Etidronate (Didronel) E. Alendronate (Fosamax)

Ibandronate, raloxifene, denosumab, and etidronate have been shown to reduce new vertebral fractures, but are not proven to prevent hip fracture. Only zoledronic acid, risedronate, and alendronate have been confirmed in sufficiently powered studies to prevent hip fracture, and these are the anti-osteoporosis drugs of choice.

When to biopsy lymph node

Immediate biopsy is warranted if the patient does not have inflammatory symptoms and the lymph node is >3 cm, if the node is in the supraclavicular area, or if the patient has coexistent constitutional symptoms such as night sweats or weight loss. Immediate evaluation is also indicated if the patient has risk factors for malignancy. Treatment with antibiotics is warranted in patients who have inflammatory symptoms such as pain, erythema, fever, or a recent infection.

You would recommend pneumococcal vaccine for which one of the following? (check one) A. A 20-year-old male who smokes 1 pack of cigarettes daily B. A 52-year-old male with type 2 diabetes mellitus who received pneumococcal vaccine 6 years ago C. A 60-year-old male who is a long-term resident of a nursing home because of a previous stroke, and who received pneumococcal vaccine at age 54 D. A 62-year-old male with chronic renal failure who received pneumococcal vaccine at age 50 and age 55 E. A 71-year-old male with no medical problems who received pneumococcal vaccine at age 65

In October 2008 the Advisory Committee on Immunization Practices (ACIP) of the Centers for Disease Control and Prevention recommended adding cigarette smoking to the list of high-risk conditions that are indications for the 23-valent pneumococcal polysaccharide vaccine. All persons between the ages of 19 and 64 who smoke should receive this vaccine. One-time revaccination after 5 years is recommended for persons with chronic renal failure, asplenia (functional or anatomic), or other immunocompromising conditions. The patient with chronic renal failure in this question has already received two immunizations. The diabetic patient and the nursing-home resident have both received one immunization and should not receive a second dose until age 65. The 71-year-old has already been immunized after age 65, and a repeat immunization is not recommended.

A 26-year-old gravida 3 para 2 was diagnosed with gestational diabetes mellitus at 24 weeks gestation. She was prescribed appropriate nutritional therapy and an exercise program. After 4 weeks, her fasting plasma glucose levels remain in the range of 105-110 mg/dL. Which one of the following would be the most appropriate treatment for this patient at this time? (check one) A. Continuation of the current regimen B. Long-acting insulin glargine (Lantus) once daily C. Pioglitazone (Actos) once daily D. A combination of intermediate-acting insulin (e.g., NPH) and a short-acting insulin (e.g., lispro) twice daily E. Sliding-scale insulin 4 times daily using ultra-short-acting insulin

In addition to an appropriate diet and exercise regimen, pharmacologic therapy should be initiated in pregnant women with gestational diabetes mellitus whose fasting plasma glucose levels remain above 100 mg/dL despite diet and exercise. There is strong evidence that such treatment to maintain fasting plasma glucose levels below 95 mg/dL and 1-hour postprandial levels below 140 mg/dL results in improved fetal well-being and neonatal outcomes. While oral therapy with metformin or glyburide is considered safe and possibly effective, insulin therapy is the best option for the pharmacologic treatment of gestational diabetes. Thiazolidinediones such as pioglitazone have not been shown to be effective or safe in pregnancy. The use of long-acting basal insulin analogues, such as glargine and detemir, has not been sufficiently evaluated in pregnancy. Sliding-scale coverage with ultra-short-acting insulin or insulin analogues, such as lispro and aspart, is generally not required in most women with gestational diabetes. While it may be effective, it involves four daily glucose checks and injections. Most patients are successfully treated with a twice-daily combination of an intermediate-acting insulin and a short-acting insulin while continuing a diet and exercise program.

A 59-year-old male with known cirrhosis is beginning to show some lower abdominal distention. Ultrasonography confirms your suspicion that he has developed moderate ascites for the first time. Which one of the following is recommended as the initial treatment of choice for this condition? (check one) A. Chlorthalidone B. Spironolactone (Aldactone) C. Furosemide (Lasix) D. Ramipril (Altace) E. Large-volume paracentesis

In patients with grade 2 ascites (visible clinically by abdominal distention, not just with ultrasonography), the initial treatment of choice is diuretics along with salt restriction. Aldosterone antagonists such as spironolactone are more effective than loop diuretics such as furosemide (SOR A). Chlorthalidone, a thiazide diuretic, is not recommended. Large-volume paracentesis is the recommended treatment of grade 3 ascites (gross ascites with marked abdominal distention), and is followed by salt restriction and diuretics.

In a patient who presents with symptoms of acute myocardial infarction, which one of the following would be an indication for thrombolytic therapy? (check one) A. New-onset ST-segment depression B. New-onset left bundle branch block C. New-onset first degree atrioventricular block D. New-onset Wenckebach second degree heart block E. Frequent unifocal ventricular ectopic beats

In patients with ischemic chest pain, the EKG is important for determining the need for fibrinolytic therapy. Myocardial infarction is diagnosed by ST elevation ≥1 mm in two or more limb leads and ≥2 mm in two or more contiguous precordial leads. In a patient with an MI, new left bundle branch block suggests occlusion of the left anterior descending artery, placing a significant portion of the left ventricle in jeopardy. Thrombolytic therapy could be harmful in patients with ischemia but not infarction - they will show ST-segment depression only. Frequent unifocal ventricular ectopy may warrant antiarrhythmic therapy, but not thrombolytic therapy.

A 42-year-old male with well-controlled type 2 diabetes mellitus presents with a 24-hour history of influenza-like symptoms, including the sudden onset of headache, fever, myalgias, sore throat, and cough. It is December, and there have been a few documented cases of influenza recently in the community. The CDC recommends initiating treatment in this situation: (check one) A. on the basis of clinical symptoms alone B. only if rapid influenza testing is positive C. only if the diagnosis is confirmed by immunoassay testing D. only if the diagnosis is confirmed by reverse transcriptase polymerase chain reaction (PCR) assay

Influenza is a highly contagious viral illness spread by airborne droplets. This patient's symptoms are highly suggestive of typical influenza: a sudden onset of malaise, myalgia, headache, fever, rhinitis, sore throat, and cough. While influenza is typically uncomplicated and self-limited, it can result in severe complications, including encephalitis, pneumonia, respiratory failure, and death. The effectiveness of treatment for influenza is dependent on how early in the course of the illness it is given. Because of the recent global H1N1 influenza outbreak that resulted in demand potentially outstripping the supply of antiviral medication, the Centers for Disease Control and Prevention has modified its recommendation as follows: Antiviral treatment is recommended as soon as possible for patients with confirmed or suspected influenza who have severe, complicated, or progressive illness or who require hospitalization. Antiviral treatment is recommended as soon as possible for outpatients with confirmed or suspected influenza who are at higher risk for influenza complications based on their age or underlying medical conditions. Clinical judgment should be an important component of outpatient treatment decisions. Antiviral treatment also may be considered on the basis of clinical judgment for any outpatient with confirmed or suspected influenza who does not have known risk factors for severe illness, if treatment can be initiated within 48 hours of illness onset. Many rapid influenza tests produce false-negative results, and more accurate assays can take more than 24 hours. Thus, treatment of patients with a clinical picture suggesting influenza is recommended, even if a rapid test is negative. Delaying treatment until further test results are available is not recommended.

A 65-year-old female who is morbidly obese presents to your office with intertrigo in the axilla. On examination you detect small, reddish-brown macules that are coalescing into larger patches with sharp borders. You suspect cutaneous erythrasma complicating the intertrigo. What would be the most appropriate topical treatment for this condition? (check one) A. Cornstarch B. A mild corticosteroid lotion C. A high-potency corticosteroid lotion D. Erythromycin

Intertrigo is inflammation of skinfolds caused by skin-on-skin friction and is common on opposing cutaneous or mucocutaneous surfaces. Secondary cutaneous bacterial and fungal infections are common complications. Cutaneous erythrasma may complicate intertrigo of interweb areas, intergluteal and crural folds, axillae, or inframammary regions. Erythrasma is caused by Corynebacterium minutissimum and presents as small reddish-brown macules that may coalesce into larger patches with sharp borders. Intertrigo complicated by erythrasma is treated with topical or oral erythromycin.

A 68-year-old female is being monitored in the hospital after elective surgery. On her third postoperative day she suddenly develops hypoxia, fever, tachycardia, and hypotension. You institute high-rate intravenous fluids and empiric antibiotics. However, approximately 2 hours into this therapy, her blood pressure remains at 80 mm Hg systolic with sluggish urine output. Which one of the following hormones should be assessed at this time? (check one) A. Aldosterone B. Catecholamines C. Cortisol D. Renin E. TSH

It has been recognized that patients suffering from a critical illness with an exaggerated inflammatory response often have a relative cortisol deficiency. Clinically, this can cause hypotension that is resistant to intravenous fluid resuscitation, and evidence is mounting that survival is increased if these patients are treated with intravenous corticosteroids during acute management. Cortisol levels can be assessed with a single serum reading, or by the change in the cortisol level after stimulation with cosyntropin (referred to as Δcortisol). The other hormones listed are not important for the acute management of a critically ill patient.

For a healthy 1-month-old, daily vitamin D intake should be: (check one) A. 50 IU B. 100 IU C. 200 IU D. 400 IU E. 800 IU

It is now recommended that all infants and children, including adolescents, have a minimum daily intake of 400 IU of vitamin D, beginning soon after birth. The current recommendation replaces the previous recommendation of a minimum daily intake of 200 IU/day of vitamin D supplementation beginning in the first 2 months after birth and continuing through adolescence. These revised guidelines for vitamin D intake for healthy infants, children, and adolescents are based on evidence from new clinical trials and the historical precedent of safely giving 400 IU of vitamin D per day in the pediatric and adolescent population. New evidence supports a potential role for vitamin D in maintaining innate immunity and preventing diseases such as diabetes mellitus and cancer.

A 67-year-old white male with hypertension and chronic kidney disease presents with the recent onset of excessive thirst, frequent urination, and blurred vision. Laboratory testing reveals a fasting blood glucose level of 270 mg/dL, a hemoglobin A 1c of 8.5%, a BUN level of 32 mg/dL, and a serum creatinine level of 2.3 mg/dL. His calculated glomerular filtration rate is 28 mL/min. Which one of the following medications should you start at this time? (check one) A. Glipizide (Glucotrol) B. Metformin (Glucophage) C. Glyburide (DiaBeta) D. Acarbose (Precose)

It is recommended that metformin be avoided in patients with a creatinine level >1.5 mg/dL for men or >1.4 mg/dL for women. Glyburide has an active metabolite that is eliminated renally. This metabolite can accumulate in patients with chronic kidney disease, resulting in prolonged hypoglycemia. Acarbose should be avoided in patients with chronic kidney disease, as it has not been evaluated in these patients. Glipizide does not have an active metabolite, and is safe in patients with chronic renal disease.

A 65-year-old white male comes to your office with a 0.5-cm nodule that has developed on his right forearm over the past 4 weeks. The lesion is dome shaped and has a central plug. You schedule a biopsy but he does not return to your office for 1 year. At that time the lesion appears to have healed spontaneously. The most likely diagnosis is (check one) A. benign lentigo B. lentigo maligna C. basal cell carcinoma D. squamous cell carcinoma E. keratoacanthoma

Keratoacanthoma grows rapidly and may heal within 6 months to a year. Squamous cell carcinoma may appear grossly and histologically similar to keratoacanthoma but does not heal spontaneously. The other lesions do not resemble keratoacanthoma.

Late decelerations on fetal monitoring are thought to indicate which one of the following? (check one) A. Fetal head compression B. Umbilical cord compression C. Fetal sleep D. Uterine hypotonus E. Uteroplacental insufficiency

Late decelerations are thought to be associated with uteroplacental insufficiency and fetal hypoxia due to decreased blood flow in the placenta. This pattern is a warning sign and is associated with increasing fetal compromise, worsening fetal acidosis, fetal central nervous system depression, and/or direct myocardial hypoxia. Early decelerations are thought to result from vagus nerve response to fetal head compression, and are not associated with increased fetal mortality or morbidity. Variable decelerations are thought to be due to acute, intermittent compression of the umbilical cord between fetal parts and the contracting uterus.

A 30-year-old white gravida 2 para 1 who has had no prenatal care presents for urgent care at 33 weeks gestation. Her symptoms include vaginal bleeding, uterine tenderness, uterine pain between contractions, and fetal distress. Her first pregnancy was uncomplicated, with a vaginal delivery at term. Which one of the following is the most likely diagnosis? (check one) A. Uterine rupture B. Vasa previa C. Placenta previa D. Placental abruption E. Cervical cancer

Late pregnancy bleeding may cause fetal morbidity and/or mortality as a result of uteroplacental insufficiency and/or premature birth. The condition described here is placental abruption (separation of the placenta from the uterine wall before delivery). There are several causes of vaginal bleeding that can occur in late pregnancy that might have consequences for the mother, but not necessarily for the fetus, such as cervicitis, cervical polyps, or cervical cancer. Even advanced cervical cancer would be unlikely to cause the syndrome described here. The other conditions listed may bring harm to the fetus and/or the mother. Uterine rupture usually occurs during active labor in women with a history of a previous cesarean section or with other predisposing factors, such as trauma or obstructed labor. Vaginal bleeding is an unreliable sign of uterine rupture and is present in only about 10% of cases. Fetal distress or demise is the most reliable presenting clinical symptom. Vasa previa (the velamentous insertion of the umbilical cord into the membranes in the lower uterine segment) is typically manifested by the onset of hemorrhage at the time of amniotomy or by spontaneous rupture of the membranes. There are no prior maternal symptoms of distress. The hemorrhage is actually fetal blood, and exsanguination can occur rapidly. Placenta previa (placental implantation that overlies or is within 2 cm of the internal cervical os) is clinically manifested as vaginal bleeding in the late second or third trimester, often after sexual intercourse. The bleeding is typically painless, unless labor or placental abruption occurs.

A 51-year-old immigrant from Vietnam presents with a 3-week history of nocturnal fever, sweats, cough, and weight loss. A chest radiograph reveals a right upper lobe cavitary infiltrate. A PPD produces 17 mm of induration, and acid-fast bacilli are present on a smear of induced sputum. While awaiting formal laboratory identification of the bacterium, which one of the following would be most appropriate? (check one) A. Observation only B. INH only C. INH and ethambutol (Myambutol) D. INH, ethambutol, and pyrazinamide E. INH, ethambutol, rifampin (Rifadin), and pyrazinamide

Leading authorities, including experts from the American Thoracic Society, CDC, and Infectious Diseases Society of America, mandate aggressive initial four-drug treatment when tuberculosis is suspected. Delays in diagnosis and treatment not only increase the possibility of disease transmission, but also lead to higher morbidity and mortality. Standard regimens including INH, ethambutol, rifampin, and pyrazinamide are recommended, although one regimen does not include pyrazinamide but extends coverage with the other antibiotics. Treatment regimens can be modified once culture results are available.

Of the following antidepressants, which one is LEAST likely to cause drug interactions? (check one) A. Citalopram (Celexa) B. Fluoxetine (Prozac) C. Paroxetine (Paxil) D. Mirtazapine (Remeron)

Like all drugs, SSRIs have significant side effects, including inhibition of the cytochrome P-450 system. However, citalopram is least likely to inhibit this system, making it a preferred SSRI for patients taking multiple medications for other illnesses

A 49-year-old female who takes multiple medications has a chemistry profile as part of her routine monitoring. She is found to have an elevated calcium level. All other values on the profile are normal, and the patient is not currently symptomatic. Follow-up testing reveals a serum calcium level of 11.2 mg/dL (N 8.4-10.2) and an intact parathyroid hormone level of 80 pg/mL (N 10-65). Which one of the following should be discontinued for 3 months before repeat laboratory evaluation and treatment? (check one) A. Lithium B. Furosemide (Lasix) C. Raloxifene (Evista) D. Calcium carbonate E. Vitamin D

Lithium therapy can elevate calcium levels by elevating parathyroid hormone secretion from the parathyroid gland. This duplicates the laboratory findings seen with mild primary hyperparathyroidism. If possible, lithium should be discontinued for 3 months before reevaluation (SOR C). This is most important for avoiding unnecessary parathyroid surgery. Vitamin D and calcium supplementation could contribute to hypercalcemia in rare instances, but they would not cause elevation of parathyroid hormone. Raloxifene has actually been shown to mildly reduce elevated calcium levels, and furosemide is used with saline infusions to lower significantly elevated calcium levels.

Hep C Risk Factors

Long term hemodialysis IV Drug Use Blood Transfusion or Organ Transplantation prior to 1992 Receipt of Clotting Factors before 1987

Osteoporosis Dx

Low impact fracture of T score < -2.5

Head Lice Tx

Malathion, Permetherin, Pyrethrins

A previously healthy 3-year-old male is brought to your office with a 4-hour history of abdominal pain followed by vomiting. Just after arriving at your office he passes bloody stool. A physical examination reveals normal vital signs, and guarding and tenderness in the right lower quadrant. A rectal examination shows blood on the examining finger. Which one of the following is the most likely diagnosis? (check one) A. Appendicitis B. Viral gastroenteritis C. Midgut volvulus D. Meckels diverticulum E. Necrotizing enterocolitis

Meckels diverticulum is the most common congenital abnormality of the small intestine. It is prone to bleeding because it may contain heterotopic gastric mucosa. Abdominal pain, distention, and vomiting may develop if obstruction has occurred, and the presentation may mimic appendicitis. Children with appendicitis have right lower quadrant pain, abdominal tenderness, guarding, and vomiting, but not rectal bleeding. With acute viral gastroenteritis, vomiting usually precedes diarrhea (usually without blood) by several hours, and abdominal pain is typically mild and nonfocal with no localized tenderness. The incidence of midgut volvulus peaks during the first month of life, but it can present anytime in childhood. Volvulus may present in one of three ways: as a sudden onset of bilious vomiting and abdominal pain in the neonate; as a history of feeding problems with bilious vomiting that now appears to be due to bowel obstruction; or, less commonly, as a failure to thrive with severe feeding intolerance. Necrotizing enterocolitis is typically seen in the neonatal intensive-care unit, occurring in premature infants in their first few weeks of life. The infants are ill, and signs and symptoms include lethargy, irritability, decreased oral intake, abdominal distention, and bloody stools. A plain abdominal film showing pneumatosis intestinalis, caused by gas in the intestinal wall, is diagnostic of this disease.

A 23-year-old Hispanic female at 18 weeks' gestation presents with a 4-week history of a new facial rash. She has noticed worsening with sun exposure. Her past medical history and review of systems is normal. On examination, you note symmetric, hyperpigmented patches on her cheeks and upper lip. The remainder of her examination is normal. The most likely diagnosis is: (check one) A. Lupus erythematosus B. Pemphigoid gestationis (herpes gestationis) C. Melasma (chloasma) D. Prurigo gestationis

Melasma or chloasma is common in pregnancy, with approximately 70% of pregnant women affected. It is an acquired hypermelanosis of the face, with symmetric distribution usually on the cheeks, nose, eyebrows, chin, and/or upper lip. The pathogenesis is not known. UV sunscreen is important, as sun exposure worsens the condition. Melasma often resolves or improves post partum. Persistent melasma can be treated with hydroquinone cream, retinoic acid, and/or chemical peels performed post partum by a dermatologist. The facial rash of lupus is usually more erythematous, and lupus is relatively rare. Pemphigoid gestationis is a rare autoimmune disease with extremely pruritic, bullous skin lesions that usually spare the face. Prurigo gestationis involves pruritic papules on the extensor surfaces and is usually associated with significant excoriation by the uncomfortable patient.

A 28-year-old male recreational runner has a midshaft posteromedial tibial stress fracture. Although he can walk without pain, he cannot run without pain. The most appropriate treatment at this point includes which one of the following? (check one) A. A short leg walking cast B. A non-weight-bearing short leg cast C. A non-weight-bearing long leg cast D. An air stirrup leg brace (Aircast) E. Low-intensity ultrasonic pulse therapy

Midshaft posteromedial tibial stress fractures are common and are considered low risk. Management consists of relative rest from running and avoiding other activities that cause pain. Once usual daily activities are pain free, low-impact exercise can be initiated and followed by a gradual return to previous levels of running. A pneumatic stirrup leg brace has been found to be helpful during treatment (SOR C). Non-weight bearing is not necessary, as this patient can walk without pain. Casting is not recommended. Ultrasonic pulse therapy has helped fracture healing in some instances, but has not been shown to be beneficial in stress fractures.

A 53-year-old male presents with a 3-month history of despondency, insomnia, and irritability with family and co-workers. During your interview you also discover that he is drinking heavily at times and has several firearms at home. He thinks his life is "useless," noting that he "would be better off dead." The most appropriate action at this time would be to: (check one) A. Prescribe an SSRI B. Arrange immediate hospitalization C. Have the patient agree to a suicide prevention contract D. Avoid direct questions regarding suicidal thoughts

More than 50% of suicides are associated with a major depressive episode and 25% are associated with a substance abuse disorder. Suicide rates increase with age and are higher among men. Increased suicide rates also occur in patients with significant medical illnesses. Because discussing suicidal ideation may relieve the patient's anxiety, the physician should directly ask depressed patients about any suicidal thoughts. There are no known reliable tools for assessing suicide risk, so the assessment is subjective. The initial management of suicidal ideation should establish safety, often by hospitalization. The suicide prevention contract is of unproven clinical and legal usefulness. Antidepressant medication has not been shown to reduce suicide rates, especially on a short-term basis

Centor Criteri and determining when to swab and stuff

Most episodes of pharyngitis are caused by viral rather than bacterial infections. The use of clinical decision rules for diagnosing group A β-hemolytic streptococcal pharyngitis improves quality of care while reducing unwarranted treatment and overall cost (SOR A). The original Centor score used four signs and symptoms to estimate the probability of acute streptococcal pharyngitis in adults with a sore throat, and was later modified by adding age as a fifth criterion. One point each is assigned for (1) absence of cough, (2) swollen, tender anterior cervical nodes, (3) temperature >38.0°C (100.4°F), and (4) a tonsillar exudate and swelling. One point is added for patients between the ages of 3 and 14 years, and a point is subtracted for patients over the age of 45. The cumulative score determines the likelihood of streptococcal pharyngitis and the need for antibiotics, and guides testing strategies. Patients with a score of zero or 1 are at very low risk for streptococcal pharyngitis and do not require testing or antibiotic therapy. Patients with a score of 2-3 should be tested using a rapid antigen test or throat culture, and a positive result warrants antibiotic therapy. Patients with a score of 4 or higher are at high risk for streptococcal pharyngitis, and empiric treatment may be considered. This patient's score is zero, and no testing or treatment is warranted.

A 36-year-old male presents with pain over the lumbar paraspinal muscles. He says the pain began suddenly while he was shoveling snow. Which one of the following is true regarding this patients injury? (check one) A. Systemic corticosteroids speed recovery B. Exercises specific to low back injuries speed recovery C. Opioids have significant advantages for symptom relief when compared with NSAIDs or acetaminophen D. Continued activity rather than bed rest helps speed recovery E. Trigger-point injections are superior to placebo in relieving acute back pain

Multiple studies have demonstrated that bed rest is detrimental to recovery from low back pain. Patients should be encouraged to remain as active as possible. Exercises designed specifically for the treatment of low back pain have not been shown to be helpful. Neither opioids nor trigger-point injections have shown superiority over placebo, NSAIDs, or acetaminophen in relieving acute back pain. There is no good evidence to suggest that systemic corticosteroids are effective for low back pain with or without sciatica

Which one of the following sleep problems in children is most likely to occur during the second half of the night? (check one) A. Confusional arousals B. Sleepwalking C. Sleep terrors D. Nightmares

Nightmares occur in the second half of the night, when rapid eye movement (REM) sleep is most prominent. Parasomnias, including sleepwalking, confusional arousal, and sleep terrors, are disorders of arousal from non-REM (NREM) sleep. These are more common in children than adults because children spend more time in deep NREM sleep. Such disorders usually occur within 1-2 hours after sleep onset, and coincide with the transition from the first period of slow-wave sleep.

An 82-year-old resident of a local nursing home is brought to your clinic with fever, difficulty breathing, and a cough productive of purulent sputum. The patient is found to have an oxygen saturation of 86% on room air and a chest radiograph shows a new infiltrate. A decision is made to hospitalize the patient. Which one of the following intravenous antibiotic regimens would be most appropriate for this patient? (check one) A. Levofloxacin (Levaquin) B. Ceftriaxone (Rocephin) and azithromycin (Zithromax) C. Ceftazidime (Fortaz, Tazicef) and levofloxacin D. Ceftazidime and vancomycin E. Ceftazidime, levofloxacin, and vancomycin

Nursing home-acquired pneumonia should be suspected in patients with a new infiltrate on a chest radiograph if it is associated with a fever, leukocytosis, purulent sputum, or hypoxia. Nursing-home patients who are hospitalized for pneumonia should be started on intravenous antimicrobial therapy, with empiric coverage for methicillin-resistant Staphylococcus aureus (MRSA) and Pseudomonas aeruginosa. The 2005 American Thoracic Society/Infectious Diseases Society of America guideline recommends combination therapy consisting of an antipseudomonal cephalosporin such as cefepime or ceftazidime, an antipseudomonal carbapenem such as imipenem or meropenem, or an extended-spectrum β-lactam/β-lactamase inhibitor such as piperacillin/tazobactam, PLUS an antipseudomonal fluoroquinolone such as levofloxacin or ciprofloxacin, or an aminoglycoside such as gentamicin, tobramycin, or amikacin, PLUS an anti-MRSA agent (vancomycin or linezolid). Ceftriaxone and azithromycin or levofloxacin alone would be reasonable treatment options for a patient with nursing home-acquired pneumonia who does not require hospitalization.

A 24-year-old female at 36 weeks' gestation plans to breastfeed her infant. She has a history of bipolar disorder, but currently is doing well without medication, and also has a history of frequent urinary tract infections. She asks you about medications that she may need to take after delivery, and how they may affect her newborn. Which one of the following would be contraindicated if she breastfeeds her infant? (check one) A. Amoxicillin B. Macrodantin (Macrobid) C. Valproic acid (Depakote) D. Lithium

Of the drugs listed, the only maternal medication that affects the infant is lithium. Breastfed infants of women taking lithium can have blood lithium concentrations that are 30%-50% of therapeutic levels. Ref: Ressel G: AAP updates statement for transfer of drugs and other chemicals into breast milk.

The mother of an 8-year-old female is concerned about purple "warts" on her daughter's hands. The mother explains that the lesions started a few months ago on the right hand along the top of most of the knuckles and interphalangeal joints, and she has recently noticed them on the left hand. The child has no other complaints and the mother denies any unusual behaviors. A physical examination is unremarkable except for the slightly violaceous, flat-topped lesions the mother described. What is the most likely cause for this patient's finger lesions? (check one) A. Dermatomyositis B. Aggressive warts C. Rubbing/wringing of the hands D. Bulimia nervosa E. Child abuse

One of the most characteristic findings in dermatomyositis is Gottron's papules, which are flat-topped, sometimes violaceous papules that often occur on most, if not all, of the knuckles and interphalangeal joints.

Which one of the following seafood poisonings requires more than just supportive treatment? (check one) A. Ciguatera B. Neurotoxic shellfish C. Paralytic shellfish D. Scombroid fish

Only symptomatic treatment is indicated for ciguatera poisoning, as there is no specific treatment. The same is true for shellfish poisoning, although potential respiratory distress or failure must be kept in mind. Scombroid poisoning is a pseudoallergic condition resulting from consumption of improperly stored scombroid fish such as tuna, mackerel, wahoo, and bonito. Nonscombroid varieties such as mahi-mahi, amberjack, sardines, and herring can also cause this problem. The poisoning is due to high levels of histamine and saurine resulting from bacterial catabolism of histidine. Symptoms occur within minutes to hours, and include flushing of the skin, oral paresthesias, pruritus, urticaria, nausea, vomiting, diarrhea, vertigo, headache, bronchospasm, dysphagia, tachycardia, and hypotension. Therapy should be the same as for allergic reactions and anaphylaxis, and will usually lead to resolution of symptoms within several hours.

A 55-year-old overweight male presents with a complaint of pain in the left big toe. He recently started jogging 2 miles a day to try to lose weight, but has not changed his diet and says he drinks 4 cans of beer every night. The pain has developed gradually over the last 2 weeks and is worse after running. An examination shows a normal foot with tenderness and swelling of the medial plantar aspect of the left first metatarsophalangeal joint. Passive dorsiflexion of the toe causes pain in that area. Plantar flexion produces no discomfort, and no numbness can be appreciated. Which one of the following is the most likely diagnosis? (check one) A. Sesamoid fracture B. Gout C. Morton's neuroma D. Cellulitis

Pain involving the big toe is a common problem. The first metatarsophalangeal (MTP) joint has two sesamoid bones, and injuries to these bones account for 12% of big-toe injuries. Overuse, a sharp blow, and sudden dorsiflexion are the most common mechanisms of injury. Gout commonly involves the first MTP joint, but the onset is sudden, with warmth, redness, and swelling, and pain on movement of the joint is common. Morton's neuroma commonly occurs between the third and fourth toes, causes numbness involving the digital nerve in the area, and usually is caused by the nerve being pinched between metatarsal heads in the center of the foot. Cellulitis of the foot is common, and can result from inoculation through a subtle crack in the skin. However, there would be redness and swelling, and the process is usually more generalized. Sesamoiditis is often hard to differentiate from a true sesamoid fracture. Radiographs should be obtained, but at times they are nondiagnostic. Treatment, fortunately, is similar, unless the fracture is open or widely displaced. Limiting weight bearing and flexion to control discomfort is the first step. More complex treatments may be needed if the problem does not resolve in 4-6 weeks.

A 50-year-old female presents with right eye pain. On examination, you find no redness, but when you test her extraocular muscles she reports marked pain with eye movement. This finding suggests that her eye pain is caused by: (check one) A. an intracranial process B. an ocular condition C. a retinal problem D. an orbital problem E. an optic nerve problem

Pain with eye movement suggests an orbital condition. Orbital inflammation, infection, or tumor invasion can lead to such eye pain. Other findings suggestive of an orbital cause of eye pain include diplopia or proptosis. If an orbital lesion is suspected, imaging studies should be performed.

Which one of the following is most characteristic of patellofemoral pain syndrome in adolescent females? (check one) A. Posterior knee pain B. Pain exacerbated by walking on a flat surface C. Inadequate hip abductor strength D. A high rate of surgical intervention

Patellofemoral pain syndrome is a common overuse injury observed in adolescent girls. The condition is characterized by anterior knee pain associated with activity. The pain is exacerbated by going up or down stairs or running in hilly terrain. It is associated with inadequate hip abductor and core strength; therefore, a prescription for a rehabilitation program is recommended. Surgical intervention is rarely required.

A 30-year-old African-American female presents with a vaginal discharge. On examination the discharge is homogeneous with a pH of 5.5, a positive whiff test, and many clue cells. Which one of the following findings in this patient is most sensitive for the diagnosis of bacterial vaginosis? (check one) A. The pH of the discharge B. The presence of clue cells C. The character of the discharge D. The whiff test

Patients must have 3 of 4 Amsel criteria to be diagnosed with bacterial vaginosis. These include a pH >4.5 (most sensitive), clue cells >20% (most specific), a homogeneous discharge, and a positive whiff test (amine odor with addition of KOH)

A 25-year-old male presents to your office with a 1-week history of neck pain with radiation to the left hand, along with intermittent numbness and tingling in the left arm. His history is negative for injury, fever, or lower extremity symptoms. Extension and rotation of the neck to the left while pressing down on the head (Spurling's maneuver) exacerbates the symptoms. His examination is otherwise normal. Cervical radiographs are negative. Which one of the following would be most appropriate at this point? (check one) A. NSAIDs for pain relief B. A trial of tricyclic antidepressants C. Cervical corticosteroid injection D. Cervical MRI E. Referral to a spine subspecialist

Patients who present with acute cervical radiculopathy and normal radiographs can be treated conservatively. The vast majority of patients with cervical radiculopathy improve without surgery. Of the interventions listed, NSAIDs are the initial treatment of choice. Tricyclic antidepressants, as well as tramadol and venlafaxine, have been shown to help with chronic neuropathic pain. Cervical MRI is not indicated unless there are progressive neurologic defects or red flags such as fever or myelopathy. Likewise, referral to a subspecialist should be reserved for patients who have persistent pain after 6-8 weeks of conservative management and for those with signs of instability. Cervical corticosteroid injections have been found to be helpful in the management of cervical radiculopathy, but should not be administered before MRI is performed (SOR C)

Which one of the following is a risk factor for endometrial cancer? (check one) A. Polycystic ovary syndrome B. Multiparity C. Late menarche D. Use of an IUD E. Use of oral contraceptives

Patients with persistent hyperestrogenic states are at heightened risk for the development of endometrial cancer. The chronic anovulation and consequent hyperstimulation of the endometrium seen with polycystic ovary syndrome predispose women to endometrial hyperplasia and carcinoma. Conversely, multiparity and late menarche are protective of the endometrium. Combination oral contraceptive use seems to decrease the risk for endometrial cancer. There is no evidence that IUD use leads to endometrial cancer, and it is thought that copper-containing IUDs may in fact provide some protection against endometrial cancer.

When added to compression therapy, which one of the following has been shown to be an effective adjunctive treatment for venous ulcers? A. Warfarin (Coumadin) B. Enoxaparin (Lovenox) C. Clopidogrel (Plavix) D. Pentoxifylline (Trental) E. Atorvastatin (Lipitor)

Pentoxifylline is effective when used with compression therapy for venous ulcers, and may be useful as monotherapy in patients unable to tolerate compression therapy. Aspirin has also been shown to be effective. Other treatments that have been studied but have not been found to be effective include oral zinc and antibiotics (SOR A).

A 74-year-old white male complains of pain in the right calf that recurs on a regular basis. He smokes 1 pack of cigarettes per day and is hypertensive. He has a history of a previous heart attack but is otherwise in fair health. Which one of the following findings would support a diagnostic impression of peripheral vascular disease? (check one) A. Pain during rest and exercise and the presence of swelling and soreness behind the knee and in the calf B. Pain that begins immediately upon walking and is unrelieved by rest C. Doppler waveform analysis showing accentuated waveforms at a point of decreased blood flow D. Treadmill arterial flow studies showing a 20-mm Hg decrease in ankle systolic blood pressure immediately following exercise E. An ankle-brachial index of 1.15

Peripheral vascular disease (PVD) is a clinical manifestation of atherosclerotic disease and is caused by occlusion of the arteries to the legs. Patients with significant arterial occlusive disease will have a prominent decrease in the ankle-brachial index from baseline following exercise, and usually a 20-mm Hg or greater decrease in systolic blood pressure. Pain during rest and exercise and the presence of swelling and soreness behind the knee and in the calf is found in those with Baker's cysts. Peripheral nerve pain commonly begins immediately upon walking and is unrelieved by rest. Doppler waveform analysis is useful in the diagnosis of PVD and will reveal attenuated waveforms at a point of decreased blood flow. Employment of the ankle-brachial index is encouraged in daily practice as a simple means to diagnose the presence of PVD. Generally, ankle-brachial indices in the range of 0.91-1.30 are thought to be normal. Ref: Hiatt WR: Medical treatment of peripheral arterial disease and claudication

A 25-year-old male who came to your office for a pre-employment physical examination is found to have 2+ protein on a dipstick urine test. You repeat the examination three times within the next month and results are still positive. Results of a 24-hour urine collection show protein excretion of <2 g/day and normal creatinine clearance. As part of his further evaluation you obtain split urine collections with a 16-hour daytime specimen containing an increased concentration of protein, and an 8-hour overnight specimen that is normal. Additional appropriate evaluation for this man's problem at this time includes which one of the following? (check one) A. Serum and urine protein electrophoresis B. Antinuclear antibody C. Serum albumin and lipid levels D. Renal ultrasonography E. No specific additional testing

Persons younger than 30 years of age who excrete less than 2 g of protein per day and who have a normal creatinine clearance should be tested for orthostatic proteinuria. This benign condition occurs in about 3%-5% of adolescents and young adults. It is characterized by increased protein excretion in the upright position, but normal protein excretion when the patient is supine. It is diagnosed using split urine collections as described in the question. The daytime specimen has an increased concentration of protein, while the nighttime specimen contains a normal concentration. Since this is a benign condition with normal renal function, no further evaluation is necessary.

You see a 17-year-old white female who has recently become sexually active. She requests oral contraceptives and you perform a brief evaluation, including blood pressure measurement. A pregnancy test is negative. She is resistant to further evaluation unless it is necessary. In addition to appropriate counseling, which one of the following should be done before prescribing oral contraceptives? (check one) A. No further evaluation at this visit unless indicated by history B. A pelvic examination and Papanicolaou test C. Screening for sexually transmitted diseases D. A breast examination

Policy statements from major organizations based on reviews of relevant medical literature support the practice of prescribing initial hormonal contraception after performing only a careful review of the medical history plus measurement of blood pressure. Requiring that patients undergo pelvic and breast examinations leads many young women to avoid this most reliable method of contraception, resulting in a much higher rate of unwanted pregnancy. Follow-up blood pressure measurements are important. Often, younger women will be willing to undergo Papanicolaou (Pap) tests and STD screening later, and periodic follow-up must be scheduled. Sexually active adolescents should have annual screening for cervical cancer and sexually transmitted diseases, but these are not necessary before prescribing oral contraceptives. The longest period of time a prescription should be given without a Pap test is 1 year, but this restriction is under study. Obviously, any history indicative of high risk would modify this approach.

A 62-year-old white male complains of fatigue and proximal extremity discomfort without any localized joint pain. Which one of the following conditions is associated with a consistently normal creatine kinase enzyme level at all phases of disease? (check one) A. Polymyalgia rheumatica B. Polymyositis C. Dermatomyositis D. Drug-induced myopathy E. Hypothyroid endocrinopathy

Polymyalgia rheumatica is a disease of the middle-aged and elderly. Discomfort is common in the neck, shoulders, and hip girdle areas. There is an absence of objective joint swelling, and findings tend to be symmetric. Characteristically, the erythrocyte sedimentation rate and C-reactive protein levels are significantly elevated; however, these tests are nonspecific. Occasionally there are mild elevations of liver enzymes, but muscle enzymes, including creatine kinase, are not elevated in this disorder. Elevation of muscle enzymes strongly suggests another diagnosis. Polymyositis and dermatomyositis are associated with variable levels of muscle enzyme elevations during the active phases of the disease. Drug-induced myopathies such as those seen with the cholesterol-lowering statin medications tend to produce some elevation of muscle enzymes during the course of the disorder. Hypothyroidism is associated with creatine kinase elevation. It should be strongly considered in the patient with unexplained, otherwise asymptomatic creatine kinase elevation found on a routine chemistry profile. Hyperthyroidism may cause muscle disease and loss of muscle, but it is not associated with creatine kinase elevation.

Which one of the following community health programs best fits the definition of secondary prevention? (check one) A. An antismoking education program at a local middle school B. Blood pressure screening at a local church C. A condom distribution program D. Screening diabetic patients for microalbuminuria

Prevention traditionally has been divided into three categories: primary, secondary, and tertiary. Primary prevention targets individuals who may be at risk to develop a medical condition and intervenes to prevent the onset of that condition (e.g., childhood vaccination programs, water fluoridation, antismoking programs, and education about safe sex). Secondary prevention targets individuals who have developed an asymptomatic disease and institutes treatment to prevent complications (e.g., routine Papanicolaou tests; screening for hypertension, diabetes, or hyperlipidemia). Tertiary prevention targets individuals with a known disease, with the goal of limiting or preventing future complications (e.g., screening diabetics for microalbuminuria, rigorous treatment of diabetes mellitus, and post-myocardial infarction prophylaxis with β-blockers and aspirin).

A 50-year-old male has a pre-employment chest radiograph showing a pulmonary nodule. There are no previous studies available. Which one of the following would raise the most suspicion that this is a malignant lesion if found on the radiograph? . (check one) A. The absence of calcification B. Location above the midline of the lung C. A diameter of 4 mm D. A solid appearance

Pulmonary nodules are a common finding on routine studies, including plain chest radiographs, and require evaluation. Radiographic features of benign nodules include a diameter <5 mm, a smooth border, a solid appearance, concentric calcification, and a doubling time of less than 1 month or more than 1 year. Features of malignant nodules include a size >10 mm, an irregular border, a "ground glass" appearance, either no calcification or an eccentric calcification, and a doubling time of 1 month to 1 year (SOR B).

Second Generation Anti-Pscyhotic Side Fx

QT Prolongation

A 79-year-old male has psychosis secondary to dementia associated with Parkinson's disease. After exhausting all other options you decide to prescribe an antipsychotic agent. Which one of the following would be the best choice in this situation? (check one) A. Haloperidol B. Olanzapine (Zyprexa) C. Risperidone (Risperdal) D. Quetiapine (Seroquel) E. Thioridazine

Quetiapine is an atypical antipsychotic that has no clinically significant effect on the dopamine D2 receptor, which is responsible for the parkinsonian side effects of antipsychotic medications. Because of this, it is considered the antipsychotic of choice in patients with dementia associated with Parkinson's disease, although its use has not been studied extensively in this clinical situation. The other atypical antipsychotics listed, olanzapine and risperidone, have some D2 receptor effect. Thioridazine and haloperidol are typical antipsychotics and have more side effects, including parkinsonian side effects; they are not recommended in this clinical situation.

What does treating H. pylori actually do?

Reduces risk of bleeding, not cancer

IBS Symptoms

Relief with defecation, diarrhea and constipation, passage of mucous, abdominal bloating

An 8-year-old male is brought to the emergency department with an acute asthma attack that began 48 hours earlier. His mother initiated his asthma action plan when the attack began, starting oral prednisolone plus albuterol (Proventil, Ventolin) by metered-dose inhaler with a spacer every 3-4 hours. In the emergency department the child is alert, with a respiratory rate of 30 beats/min and an oxygen saturation of 94% on room air. He is audibly wheezing. Peak flow is 40% of the predicted value. (check one) A. Continue the current albuterol treatment but switch to a nebulizer B. Administer high-dose albuterol via nebulizer every 20 minutes for 1 hour C. Administer intravenous corticosteroids within the first hour D. Administer magnesium sulfate intravenously E. Prescribe high-dose mucolytics and chest physiotherapy

Repeated doses of a short-acting β2-agonist and correction of hypoxia are the main elements of initial emergency department treatment for acute asthma exacerbations in children. Nebulizer treatments are no better than a metered-dose inhaler with a spacer (SOR A). High-dose nebulized albuterol every 20 minutes for 1 hour has not been shown to be beneficial. In children already receiving standard treatment with albuterol and corticosteroids the addition of intravenous magnesium sulfate has been shown to improve lung function and reduce the need for hospitalization (SOR A). Oral administration of corticosteroids is as effective as the intravenous route for reducing the need for hospital admission (SOR A). Mucolytics and chest physiotherapy have not been shown to be effective in children with acute asthma attacks.

You see a 68-year-old mechanic for a routine evaluation. He has a 2-year history of hypertension. His weight is normal and he adheres to his medication regimen. His current medications are metoprolol (Lopressor), 100 mg twice daily; olmesartan (Benicar), 40 mg/day; and hydrochlorothiazide, 25 mg/day. His serum glucose levels have always been normal, but his lipid levels are elevated. A physical examination is unremarkable except for an enlarged prostate and a blood pressure of 150/94 mm Hg. Laboratory studies show a serum creatinine level of 1.6 mg/dL (N 0.6-1.5) and a serum potassium level of 4.9 mmol/L (N 3.5-5.0). The patient's record shows blood pressures ranging from 145/80 mm Hg to 148/96 mm Hg over the past year. Which one of the following would be most appropriate at this point? (check one) A. Continue his current management with no changes B. Substitute furosemide (Lasix) for hydrochlorothiazide C. Add clonidine (Catapres) D. Add spironolactone (Aldactone) E. Add hydralazine (Apresoline)

Resistant or refractory hypertension is defined as a blood pressure ≥140/90 mm Hg, or ≥130/80 mm Hg in patients with diabetes mellitus or renal disease (i.e., with a creatinine level >1.5 mg/dL or urinary protein excretion >300 mg over 24 hours), despite adherence to treatment with full doses of at least three antihypertensive medications, including a diuretic. JNC 7 guidelines suggest adding a loop diuretic if serum creatinine is >1.5 mg/dL in patients with resistant hypertension.

Which one of the following side effects induced by traditional neuroleptic agents responds to treatment with beta-blockers? (check one) A. Akathisia B. Rigidity C. Dystonia D. Sialorrhea E. Stooped posture

Rigidity, sialorrhea, and stooped posture are parkinsonian side effects of neuroleptic drugs. These are treated with anticholinergic drugs such as benztropine or amantadine. Dystonia, often manifested as an acute spasm of the muscles of the head and neck, also responds to anticholinergics. Akathisia (motor restlessness and an inability to sit still) can be treated with either anticholinergic drugs or beta-blockers.

Osteoporotic bone loss can be caused or accelerated by prolonged use of which one of the following medications? (check one) A. Hydrochlorothiazide B. Phenytoin C. Raloxifene (Evista) D. Diazepam (Valium) E. Fluoxetine (Prozac)

Secondary osteoporosis can result from a variety of endocrine, nutritional, or genetic disorders, as well as from prolonged use of certain medications. Anticonvulsants such as phenytoin increase the hepatic metabolism of vitamin D, thereby reducing intestinal calcium absorption. Other medications that adversely affect bone mineral density include glucocorticoids, cyclosporine, phenobarbital, and heparin. Thiazide diuretics reduce urinary calcium loss and are believed to preserve bone density with long-term use. Benzodiazepines and SSRIs have not been associated with increases in bone loss or in hip fractures. Raloxifene, a selective estrogen receptor modulator, is indicated for the prevention and treatment of osteoporosis in postmenopausal women.

In the secondary prevention of ischemic cardiac events, which one of the following is most likely to be beneficial in a 68-year-old female with known coronary artery disease and preserved left ventricular function? (check one) A. ACE inhibitors B. Hormone therapy C. Calcium channel blockers D. Vitamin E E. Oral glycoprotein IIb/IIIa receptor inhibitors

Secondary prevention of cardiac events consists of long-term treatment to prevent recurrent cardiac morbidity and mortality in patients who have either already had an acute myocardial infarction or are at high risk because of severe coronary artery stenosis, angina, or prior coronary surgical procedures. Effective treatments include aspirin, β-blockers after myocardial infarction, ACE inhibitors in patients at high risk after myocardial infarction, angiotensin II receptor blockers in those with coronary artery disease, and amiodarone in patients who have had a myocardial infarction and have a high risk of death from cardiac arrhythmias. Oral glycoprotein IIb/IIIa receptor inhibitors appear to increase the risk of mortality when compared with aspirin. Calcium channel blockers, class I anti-arrhythmic agents, and sotalol all appear to increase mortality compared with placebo in patients who have had a myocardial infarction. Contrary to decades of large observational studies, multiple randomized, controlled trials show no cardiac benefit from hormone therapy in postmenopausal women.

Which one of the following is recommended in the treatment of all four stages of COPD, from mild through very severe? (check one) A. Scheduled oral mucolytics B. Scheduled inhaled corticosteroids such as fluticasone (Flovent HFA) C. Scheduled long-acting inhaled bronchodilators such as salmeterol (Serevent) D. Scheduled long-acting anticholinergics such as tiotropium (Spiriva) E. Short-acting inhaled ß2 -agonists such as albuterol (Ventolin HFA), as needed for dyspnea

Short-acting bronchodilators such as albuterol and ipratropium are recommended on an as-needed basis for treatment of breathlessness in stage I (mild) COPD. They are also recommended for as-needed use in stage II (moderate), stage III (severe), and stage IV (very severe) COPD. Long-acting bronchodilators such as salmeterol or tiotropium are recommended for stages II, III, and IV. Inhaled corticosteroids are recommended for stages III and IV. Mucolytics can be considered for stages III and IV.

Metformin (Glucophage) should be stopped prior to which one of the following, and withheld until 48 hours after completion of the test? (check one) A. An upper GI series B. Abdominal ultrasonography C. CT angiography D. MRI of the brain E. Colonoscopy

Since even a temporary reduction in renal function, such as occurs after pyelography or angiography, can cause lactic acidosis in patients taking metformin, the drug should be discontinued 48 hours before such procedures (SOR C) and restarted 48 hours after the procedure if renal function is normal. The other procedures listed are not indications for stopping metformin.

A 75-year-old Hispanic male presents with dyspnea on exertion which has worsened over the last several months. He denies chest pain and syncope, and was fairly active until the shortness of breath slowed him down recently. You hear a grade 3/6 systolic ejection murmur at the right upper sternal border which radiates into the neck. Echocardiography reveals aortic stenosis, with a mean transvalvular gradient of 55 mm Hg and a calculated valve area of 0.6 cm2. Left ventricular function is normal. Which one of the following is appropriate management for this patient? (check one) A. Aortic valve replacement B. Aortic balloon valvotomy C. Medical management with beta-blockers and nitrates D. Watchful waiting until the gradient is severe enough for treatment E. Deferring the decision pending results of an exercise stress test

Since this patient's mean aortic-valve gradient exceeds 50 mm Hg and the aortic-valve area is not larger than 1 cm2, it is likely that his symptoms are due to aortic stenosis. As patients with symptomatic aortic stenosis have a dismal prognosis without treatment, prompt correction of his mechanical obstruction with aortic valve replacement is indicated. Medical management is not effective, and balloon valvotomy only temporarily relieves the symptoms and does not prolong survival. Patients who present with dyspnea have only a 50% chance of being alive in 2 years unless the valve is promptly replaced. Exercise testing is unwarranted and dangerous in patients with symptomatic aortic stenosis.

A 50-year-old Hispanic male has a solitary 5-mm pulmonary nodule on a chest radiograph. His only medical problem is severe osteoarthritis. He quit smoking 10 years ago. Which one of the following would be the most appropriate follow-up for the pulmonary nodule? (check one) A. Positron emission tomography (PET) B. Chest CT C. A repeat chest radiograph in 6 weeks D. A repeat chest radiograph in 6 months

Solitary pulmonary nodules are common radiologic findings, and the differential diagnosis includes both benign and malignant causes. The American College of Chest Physicians guidelines for evaluation of pulmonary nodules are based on size and patient risk factors for cancer. Lesions ≥8 mm in diameter with a "ground-glass" appearance, an irregular border, and a doubling time of 1 month to 1 year suggest malignancy, but smaller lesions should also be evaluated, especially in a patient with a history of smoking. CT is the imaging modality of choice to reevaluate pulmonary nodules seen on a radiograph (SOR C). PET is an appropriate next step when the cancer pretest probability and imaging results are discordant (SOR C). Patients with notable nodule growth during follow-up should undergo a biopsy (SOR C).`

A 25-year-old white female comes to your office complaining of abdominal pain. She requests that you hospitalize her and do whatever is necessary to get rid of the pain that has been present for a number of years. She has difficulty describing the pain. She is a single parent, and becomes defensive when asked about her previous marriage, stating only that her former husband is an alcoholic, "just like my father." Her previous medical history includes an appendectomy, a cholecystectomy, and a hysterectomy. On physical examination she appears healthy and a CBC, erythrocyte sedimentation rate, serum amylase level, serum electrolyte levels, and multiple chemical profile are all normal. Management of this patient should include which one of the following? (check one) A. Long-term use of antidepressants B. Referral to a surgeon for exploratory laparotomy C. Informing her that her problems are psychogenic and that there is nothing to worry about D. Hospitalization as requested, then consultation with a psychiatrist E. Scheduling frequent, regular office visits

Somatoform disorder is often encountered in family practice. Studies have documented that 5% of patients meet the criteria for somatization disorder, while another 4% have borderline somatization disorder. Most of these patients are female and have a low socioeconomic status. They have a high utilization of medical services, usually reflected by a thick medical chart, and are often single parents. As a rule, physicians tend to be less satisfied with the care rendered to these patients as opposed to those without the disorder. Patients with multiple unexplained physical complaints have been described as functionally disabled, spending an average of one week per month in bed. Many of these patients seek and are ultimately granted surgical procedures, and it is not uncommon for them to have multiple procedures, especially involving the pelvic area. Often there are associated psychiatric symptoms such as anxiety, depression, suicidal threats, alcohol or drug abuse, interpersonal or occupational difficulties, and antisocial behavior. A background of a dysfunctional family unit in which one or both parents abused alcohol or drugs or were somatically preoccupied is also quite common. Unfortunately, these individuals tend to marry alcohol abusers, and thus continue the pattern of dysfunctional family life. Treatment of somatoform disorder should be by one primary physician where an established relationship and regular visits can curtail the dramatic symptoms that many times lead to hospitalization. The family physician is in a position to monitor family dynamics and provide direction on such issues as alcoholism and child abuse. Each office visit should be accompanied by a physical examination, and the temptation to tell the patient that the problem is not physical should be avoided. Knowing the patient well helps to avoid unnecessary hospitalization, diagnostic procedures, surgery, and laboratory tests. These should be done only if clearly indicated. Psychotropic medications should be avoided except when clearly indicated, as medications reinforce the sick role, may be abused, and may be used for suicidal gestures. Following these recommendations significantly decreases the cost of care for the patient.

Which one of the following would suggest that the sudden and unexpected death of a healthy infant resulted from deliberate suffocation rather than sudden infant death syndrome? (check one) A. No previous history of apneic episodes B. An age of 9 months C. Mottled skin D. Clenched fists E. Blood-tinged froth in the mouth

Sudden infant death syndrome (SIDS) is the most common cause of death during the first 6 months of life in the United States, with a peak incidence at 2-4 months of age and a quick dropoff by the age of 6 months. The cause of death is a retrospective diagnosis of exclusion, and is supported by a history of quiet death during sleep in a previously healthy infant younger than 6 months of age. Evidence of terminal activity may be present, such as clenched fists or a serosanguineous, blood-tinged, or mucoid discharge from the mouth or nose. Lividity and mottling are frequently present in dependent areas.

A 62-year-old male with a history of prostate cancer and well-controlled hypertension presents with severe osteoporosis. At 55 years of age he received prostate brachytherapy and androgen deprivation for his prostate cancer and has been disease-free since. He presently takes lisinopril (Prinivil, Zestril), 5 mg daily; alendronate (Fosamax), 70 mg weekly; calcium, 1000 mg daily; and vitamin D, 1200 units daily. He has never smoked, exercises five times a week, and maintains a healthy lifestyle. In spite of his lifestyle and the medications he takes, he continues to have severe osteoporosis on his yearly bone density tests. In addition to recommending fall precautions, which one of the following would you consider next to treat his osteoporosis? (check one) A. Testosterone B. Calcitonin C. Teriparatide (Forteo) D. Raloxifene (Evista) E. Zoledronic acid (Reclast)

Teriparatide is indicated for the treatment of severe osteoporosis, for patients with multiple osteoporosis risk factors, or for patients with failure of bisphosphonate therapy (SOR B). Therapy with teriparatide is currently limited to 2 years and is contraindicated in patients with a history of bone malignancy, Paget disease, hypercalcemia, or previous treatment with skeletal radiation. Its route of administration (subcutaneous) and high cost should be considered when prescribing teriparatide therapy. Testosterone therapy is contraindicated in patients with a history of prostate cancer. Zoledronic acid is a parenterally administered bisphosphonate and would not be appropriate in a patient who has already failed bisphosphonate therapy. Likewise, raloxifene and calcitonin are not indicated in patients with severe osteoporosis who have failed bisphosphonate therapy.

A critically ill adult male is admitted to the intensive-care unit because of sepsis. He has no history of diabetes mellitus, but his glucose level on admission is 215 mg/dL and insulin therapy is ordered. Which one of the following is the most appropriate target glucose range for this patient? (check one) A. 80-120 mg/dL B. 100-140 mg/dL C. 120-160 mg/dL D. 140-180 mg/dL E. 160-200 mg/dL

The 2009 consensus guidelines on inpatient glycemic control issued by the American Association of Clinical Endocrinologists and the American Diabetes Association recommend insulin infusion with a target glucose level of 140-180 mg/dL in critically ill patients. This recommendation is based on clinical trials in critically ill patients. In the groups studied, there was no reduction in mortality from intensive treatment targeting near-euglycemic glucose levels compared to conventional management with a target glucose level of <180 mg/dL. There also were reports of harm resulting from intensive glycemic control, including higher rates of severe hypoglycemia and even increased mortality.

The mother of a 3-year-old male is concerned that he doesn't like being held, doesn't interact much with other children, and rarely smiles. Of the following, which feature would be most helpful in distinguishing Asperger's syndrome from autism in this patient? (check one) A. Normal language development B. Delayed gross motor development C. Repetitive fine motor mannerisms D. Preoccupation with parts of objects E. Focused patterns of intense interest

The DSM-IV categorizes Asperger's syndrome and autism as pervasive developmental disorders. In both conditions, children have significant difficulties with social interactions, although the impairment is more severe and sustained in autism. Both Asperger's and autism may be associated with symptoms of repetitive motor mannerisms, restricted patterns of interest (which are abnormal in focus or intensity), or preoccupation with parts of objects. However, unlike children with Asperger's syndrome, autistic children have serious problems with communication skills, either in the development of speech itself or in the ability to carry on a conversation. Normal, age-appropriate language skills in a 3-year-old would rule out a diagnosis of autism. It is an important distinction to make, as the prognosis for independent functioning in children with Asperger's syndrome is significantly better than in children with autism.

Which one of the following patients should be advised to take aspirin, 81 mg daily, for the primary prevention of stroke? (check one) A. A 42-year-old male with a history of hypertension B. A 72-year-old female with no chronic medical conditions C. An 80-year-old male with a history of depression D. An 87-year-old female with a history of peptic ulcer disease `

The U.S. Preventive Services Task Force (USPSTF) has summarized the evidence for the use of aspirin in the primary prevention of cardiovascular disease as follows: The USPSTF recommends the use of aspirin for men 45-79 years of age when the potential benefit from a reduction in myocardial infarctions outweighs the potential harm from an increase in gastrointestinal hemorrhage (Grade A recommendation) The USPSTF recommends the use of aspirin for women 55-79 years of age when the potential benefit of a reduction in ischemic strokes outweighs the potential harm of an increase in gastrointestinal hemorrhage (Grade A recommendation) The USPSTF concludes that the current evidence is insufficient to assess the balance of benefits and harms of aspirin for cardiovascular disease prevention in men and women 80 years of age or older (Grade I statement) The USPSTF recommends against the use of aspirin for stroke prevention in women younger than 55 and for myocardial infarction prevention in men younger than 45 (Grade D recommendation) In summary, consistent evidence from randomized clinical trials indicates that aspirin use reduces the risk for cardiovascular disease events in adults without a history of cardiovascular disease. It reduces the risk for myocardial infarction in men, and ischemic stroke in women. Consistent evidence shows that aspirin use increases the risk for gastrointestinal bleeding, and limited evidence shows that aspirin use increases the risk for hemorrhagic strokes. The overall benefit in the reduction of cardiovascular disease events with aspirin use depends on baseline risk and the risk for gastrointestinal bleeding.

A 12-year-old white male is brought to your office after accidentally cutting his left hand with a pocketknife. On examination you find a deep 2-cm laceration at the base of the thenar eminence. To test for motor injury to the median nerve you would have the patient: (check one) A. Extend the thumb and fingers B. Oppose the thumb and little finger C. Flex the wrist D. Abduct the thumb and index finger

The ability to touch the tip of the thumb to the tip of the little finger indicates normal motor function of the median nerve. The radial nerve controls extension of the thumb and fingers. The median nerve partially controls flexion of the wrist, but the site of innervation is proximal to the wound site at the base of the thumb. Abduction of the thumb is a function of the radial nerve. Finger abduction is a function of the ulnar nerve.

You are consulted for medical management of a 45-year-old male, previously unknown to you, who is hospitalized in the psychiatric unit with paranoid schizophrenia. His fasting blood glucose level is 180 mg/dL. Which one of the following medications is the most likely cause of the hyperglycemia? (check one) A. Alprazolam (Xanax) B. Haloperidol (Haldol) C. Chlorpromazine (Thorazine) D. Olanzapine (Zyprexa) E. Thiothixene (Navane)

The atypical antipsychotics include clozapine, olanzapine, risperidone, ziprasidone, quetiapine, and aripiprazole. As a class, they have fewer extrapyramidal side effects than the classical antipsychotics haloperidol, thiothixene, chlorpromazine, and others. Some of the atypical agents, notably olanzapine and clozapine, have been associated with hyperglycemia and the development of type 2 diabetes mellitus. Neither benzodiazepines like alprazolam nor the classical antipsychotics have been associated with hyperglycemia.

The FDA has imposed a black box warning on all thiazolidinediones, such as pioglitazone (Actos). This warning addresses a contraindication to the prescription of these drugs in patients with: (check one) A. renal insufficiency B. dementia C. exposure to radiocontrast media D. heart failure E. respiratory failure

The black box warning for thiazolidinediones specifically addresses heart failure. These agents are also contraindicated in patients with type 1 diabetes mellitus or hepatic disease, and in premenopausal anovulatory women

Which cardiac arrhythmia has been reported with high-dose methadone use? (check one) A. Atrial fibrillation B. Paroxysmal supraventricular tachycardia C. Third degree heart block D. Torsades de pointes E. Multifocal atrial tachycardia

The cardiac toxicity of methadone is primarily related to QT prolongation and torsades de pointes

You have provided care for a 27-year-old married, monogamous female for several years. One year ago, she had abnormal cervical cytology that was interpreted as "atypical squamous cells of undetermined significance" (ASC-US). She had repeat cervical cytologic examinations 6 months ago and again last week, both reported as negative. Which one of the following would be the most appropriate next step? (check one) A. Repeat cervical cytology again in 4-6 months B. Screening for human papillomavirus C. Colposcopic examination with a biopsy and endocervical curettage D. Resuming a routine screening protocol E. Cervical culture for herpesvirus

The cervical cytology category of atypical squamous cells of undetermined significance (ASC-US) is one that is poorly reproducible, having been shown to be frequently downgraded to negative or upgraded to a low- or high-grade squamous intraepithelial lesion on review. Recommended management strategies for women with ASC-US include repeat cytology at 4-6 months, immediate colposcopy, and reflex DNA testing for oncogenic HPV types. Should two repeat cytologic examinations at 4- to 6-month intervals prove negative, the patient can safely return to routine cytologic screenings. Should any repeat examination detect ASC-US or more significant cytology, colposcopy is indicated

An otherwise healthy 37-year-old male presents to your office with a 2-week history of redness and slight irritation in his groin. On examination a tender erythematous plaque with mild scaling is seen in his right crural fold. The area fluoresces coral-red under a Wood's light. Which one of the following would be the most appropriate treatment at this time? (check one) A. Amoxicillin B. Erythromycin C. Ketoconazole D. Nystatin (Mycostatin) E. Triamcinolone (Kenalog)

The characteristics of this lesion, including coral-red fluorescence under a Wood's light, suggests Corynebacterium infection, which is associated with erythrasma. Tinea cruris caused by Microsporum infection fluoresces green, while intertrigo and tinea cruris caused by Epidermophyton or Trichophyton infections do not fluoresce. Erythromycin, either systemic or topical, is the treatment of choice.

A 40-year-old female comes to your office for a routine examination. She has been in good health and has no complaints other than obesity. Her mother is diabetic and the patient has had a child that weighed 9 lb at birth. Her examination is negative except for her obesity. A fasting glucose level is 128 mg/dL, and when repeated 2 days later it is 135 mg/dL. Which one of the following would be most appropriate at this point? (check one) A. Diagnose type 2 diabetes mellitus and begin diet and exercise therapy B. Begin an oral hypoglycemic agent C. Order a glucose tolerance test D. Tell the patient that she has impaired glucose homeostasis but is not diabetic

The criteria for diagnosing diabetes mellitus include any one of the following: symptoms of diabetes (polyuria, polydipsia, weight loss) plus a casual glucose level ≥200 mg/dL; a fasting plasma glucose level ≥126 mg/dL; or a 2-hour postprandial glucose level ≥200 mg/dL after a 75 gram glucose load. In the absence of unequivocal hyperglycemia the test must be repeated on a different day. The criteria for impaired glucose homeostasis include either a fasting glucose level of 100-125 mg/dL (impaired fasting glucose) or a 2-hour glucose level of 140-199 mg/dL on an oral glucose tolerance test. Normal values are now considered <100 mg/dL for fasting glucose and <140 mg/dL for the 2-hour glucose level on an oral glucose tolerance test.

A 15-month-old male is brought to your office 3 hours after the onset of an increased respiratory rate and wheezing. He has an occasional cough and no rhinorrhea. His immunizations are up to date and he attends day care regularly. His temperature is 38.2°C (100.8°F), respiratory rate 42/min, and pulse rate 118 beats/min. The child is sitting quietly on his mother's lap. His oxygen saturation is 94% on room air. On examination you note inspiratory crackles in the left lower lung field. The child appears to be well hydrated and the remainder of the examination, including an HEENT examination, is normal. Nebulized albuterol (AccuNeb) is administered and no improvement is noted. Which one of the following would be most appropriate in the management of this patient? (check one) A. Laboratory evaluation B. Inpatient monitoring, with no antibiotics at this time C. Hospitalization and intravenous ceftriaxone (Rocephin) D. Close outpatient follow-up, with no antibiotics at this time E. Oral high-dose amoxicillin (90 mg/kg/day), with close outpatient follow-up

The diagnosis of community-acquired pneumonia is mostly based on the history and physical examination. Pneumonia should be suspected in any child with fever, cyanosis, and any abnormal respiratory finding in the history or physical examination. Children under 2 years of age who are in day care are at higher risk for developing community-acquired pneumonia. Laboratory tests are rarely helpful in differentiating viral versus bacterial etiologies and should not be routinely performed. Outpatient antibiotics are appropriate if the child does not have a toxic appearance, hypoxemia, signs of respiratory distress, or dehydration. Streptococcus pneumoniae is one of the most common etiologies in this age group, and high-dose amoxicillin is the drug of choice.

A 50-year-old male is brought to the emergency department with shortness of breath, chest tightness, tremulousness, and diaphoresis. Aside from tachypnea, the physical examination is normal. Arterial blood gases on room air show a pO2 of 98 mm Hg (N 80-100), a pCO2 of 24 mm Hg (N 35-45), and a pH of 7.57 (N 7.38-7.44). The most likely cause of the patient's blood gas abnormalities is: (check one) A. carbon monoxide poisoning B. anxiety disorder with hyperventilation C. an acute exacerbation of asthma D. pulmonary embolus E. pneumothorax

The elevated pH, normal oxygen saturation, and low pCO2 are characteristic of acute respiratory alkalosis, as seen with acute hyperventilation states. In patients with a pulmonary embolism, pO2 and pCO2 are decreased, while the pH is elevated, indicating the acute nature of the disorder. With the other diagnoses, findings on the physical examination would be different than those seen in this patient. Vital signs would be normal with carbon monoxide poisoning, and patients with an asthma exacerbation have a prominent cough and wheezing, and possibly other abnormalities. Tension pneumothorax causes severe cardiac and respiratory distress, with significant physical findings including tachycardia, hypotension, and decreased mental activity.

A 3-year-old toilet-trained female is brought to your office by her mother, who has noted a red rash on the child's perineum for the last 5 days. The rash is pruritic and has been spreading. The mother has treated the area for 3 days with nystatin cream with no obvious improvement. The child has not used any other recent medications and has no significant past medical history. Your examination reveals a homogeneous, beefy red rash surrounding the vulva and anus. The most likely etiologic agent is: (check one) A. Malassezia furfur B. Escherichia coli C. Haemophilus influenzae D. Staphylococcus aureus E. group A Streptococcus pyogenes

The epidemiology of group A streptococcal disease of the perineum is similar to that of group A streptococcal pharyngitis, and the two often coexist. It is theorized that either auto-inoculation from mouth to hand to perineum occurs, or that the bacteria is transmitted through the gastrointestinal tract. In one study, the average age of patients with this disease varied from 1 to 11 years, with a mean of 5 years. Girls and boys were almost equally affected. The incidence is estimated to be about 1 in 200 pediatric visits and peaks in March, April, and May in North America. The condition usually presents with itching and a beefy redness around the anus and/or vulva and will not clear with medications used to treat candidal infections.

A 28-year-old female presents for evaluation of a persistent thin discharge, with a "fishy" odor particularly noticeable after intercourse. She has no dyspareunia or dysuria, is in a monogamous relationship, and has used oral contraceptives for many years. Physical examination reveals no vulvar, vaginal, or cervical erythema. There is a homogenous white discharge that coats the vaginal walls. The vaginal pH is 7.0 and on microscopy you note stippled epithelial cells but no hyphae or trichomonads. Which one of the following is true regarding this patient? (check one) A. The treatment of choice may interact with alcohol B. The patient's partner needs to be treated simultaneously C. The diagnosis should be confirmed with a culture D. Oral contraceptives contribute to the risk for this condition

The patient has the typical symptoms and signs of bacterial vaginosis. There is no need for confirmatory testing. The treatment of choice is oral metronidazole, which may cause a disulfiram-like interaction with alcohol. Treatment of the partner has not been shown to improve the outcome.

A 30-year-old female who had a deep venous thrombosis in her left leg during pregnancy has an uneventful delivery. During the pregnancy she was treated with low molecular weight heparin. Just after delivery her left leg is pain free and is not swollen. She plans to resume normal activities soon. Which one of the following would be most appropriate with regard to anticoagulation? (check one) A. Discontinuing treatment, with no further evaluation B. Discontinuing treatment if venous Doppler ultrasonography is negative for thrombus C. Continuing low molecular weight heparin for 6 more weeks D. Switching to low-dose unfractionated heparin for 6 weeks E. Switching to aspirin for 6 weeks

The risk of pulmonary embolism continues in the postpartum period, and may actually increase during that time. For patients who have had a deep-vein thrombosis during pregnancy, treatment should be continued for 6 weeks after delivery, with either warfarin or low molecular weight heparin.

Which one of the following platelet counts is the threshold for prophylactic platelet transfusion in most patients? (check one) A. 10,000/μL B. 25,000/μL C. 40,000/μL D. 50,000/μL E. 100,000/μL

The threshold for prophylactic platelet transfusion is 10,000/μL (SOR A). Platelet transfusion decreases the risk of spontaneous bleeding in such patients. A count below 50,000/μL is an indication for platelet transfusion in patients undergoing an invasive procedure.

A 5-month-old female is brought in with a 1-day history of an axillary temperature of 100.6°F and mild irritability. Findings are normal on examination except for a runny nose and a moderately distorted, immobile, red right eardrum. There is no history of recent illness or otitis in the past. The most appropriate management would be: (check one) A. azithromycin (Zithromax) for 5 days B. amoxicillin for 10 days C. amoxicillin for 5 days D. oral decongestants E. observation and a repeat examination in 2 weeks

The treatment for otitis media is evolving. Recommendations by the American Academy of Family Physicians and the American Academy of Pediatrics advocate a 10-day course of antibiotics for children under the age of 2 years if the diagnosis is certain. If the diagnosis is not certain and the illness is not severe, there is an option of observation with follow-up. For children over the age of 2 years, the recommendation is still to treat if the diagnosis is certain, but there is an option of observation and follow-up if the illness is not severe and follow-up can be guaranteed. Amoxicillin is the first-line therapy; the recommended dosage is 80-90 mg/kg/day in two divided doses, which increases the concentration of amoxicillin in the middle ear fluid to help with resistant Pneumococcus. Azithromycin, because of a broader spectrum and potential for causing resistance, is not considered the treatment of first choice. Treatment regimens ranging from 5 to 7 days are appropriate for selected children over the age of 5 years.

A 12-year-old white male asthmatic has an acute episode of wheezing. You diagnose an acute asthma attack and prescribe an inhaled β2-adrenergic agonist. After 2 hours of treatment, he continues to experience wheezing and shortness of breath. Which one of the following is the most appropriate addition to acute outpatient management? (check one) A. Oral theophylline (Theo-Dur) B. Oral corticosteroids C. An oral β-adrenergic agonist D. Inhaled cromolyn (Intal) E. Inhaled corticosteroids

The treatment of choice for occasional acute symptoms of asthma is an inhaled β2-adrenergic agonist such as albuterol, terbutaline, or pirbuterol. If symptoms do not respond to β-agonists, they should be treated with a short course of systemic corticosteroids. Theophylline has limited usefulness for treatment of acute symptoms in patients with intermittent asthma; it is a less potent bronchodilator than subcutaneous or inhaled adrenergic drugs, and therapeutic serum concentrations can cause transient adverse effects such as nausea and central nervous system stimulation in patients who have not been taking the drug continuously. Cromolyn can decrease airway hyperreactivity, but has no bronchodilating activity and is useful only for prophylaxis. Inhaled corticosteroids should be used to suppress the symptoms of chronic persistent 2 asthma. Oral β2-selective agonists are less effective and have a slower onset of action than the same drugs given by inhalation.

Which one of the following is an absolute contraindication to electroconvulsive therapy (ECT)? (check one) A. Age >80 years B. A cardiac pacemaker C. An implantable cardioverter-defibrillator D. Pregnancy E. There are no absolute contraindications to ECT

There are no absolute contraindications to electroconvulsive therapy (ECT), but factors that have been associated with reduced efficacy include a prolonged episode, lack of response to medication, and coexisting psychiatric diagnoses such as a personality disorder. Persons who may be at increased risk for complications include those with unstable cardiac disease such as ischemia or arrhythmias, cerebrovascular disease such as recent cerebral hemorrhage or stroke, or increased intracranial pressure. ECT can be used safely in elderly patients and in persons with cardiac pacemakers or implantable cardioverter-defibrillators. ECT also can be used safely during pregnancy, with proper precautions and in consultation with an obstetrician.

Patients with which one of the following conditions are at increased risk for complications from electroconvulsive therapy for depression? (check one) A. Pregnancy B. Seizure disorder C. Cardiac pacemaker implantation D. Depression unresponsive to oral medications E. Recent cerebral hemorrhage

There are no absolute contraindications to electroconvulsive therapy (ECT), but more complications are seen in patients with a history of recent cerebral hemorrhage, stroke, or increased intracranial pressure. The efficacy of ECT may be reduced in patients who have not responded to oral antidepressants

An asymptomatic 3-year-old male presents for a routine check-up. On examination you notice a systolic heart murmur. It is heard best in the lower precordium and has a low, short tone similar to a plucked string or kazoo. It does not radiate to the axillae or the back and seems to decrease with inspiration. The remainder of the examination is normal. Which one of the following is the most likely diagnosis? (check one) A. Eisenmenger's syndrome B. Mitral stenosis C. Peripheral pulmonic stenosis D. Still's murmur E. Venous hum

There are several benign murmurs of childhood that have no association with physiologic or anatomic abnormalities. Of these, Still's murmur best fits the murmur described. The cause of Still's murmur is unknown, but it may be due to vibrations in the chordae tendinae, semilunar valves, or ventricular wall. A venous hum consists of a continuous low-pitched murmur caused by the collapse of the jugular veins and their subsequent fluttering, and it worsens with inspiration or diastole. The murmur of physiologic peripheral pulmonic stenosis (PPPS) is caused by physiologic changes in the newborns pulmonary vessels. PPPS is a systolic murmur heard loudest in the axillae bilaterally that usually disappears by 9 months of age. Mitral stenosis causes a diastolic murmur, and Eisenmenger's syndrome involves multiple abnormalities of the heart that cause significant signs and symptoms, including shortness of breath, cyanosis, and organomegaly, which should become apparent from a routine history and examinatio`

The probability of pregnancy after unprotected intercourse is the highest at which one of the following times? (check one) A. 3 days before ovulation B. 1 day before ovulation C. The day of ovulation D. 1 day after ovulation E. 3 days after ovulation

There is a 30% probability of pregnancy resulting from unprotected intercourse 1 or 2 days before ovulation, 15% 3 days before, 12% the day of ovulation, and essentially 0% 1-2 days after ovulation. Knowing the time of ovulation therefore has implications not only for "natural" family planning, but also for decisions regarding postcoital contraception

Which one of the following is necessary to make a diagnosis of polymyalgia rheumatica? (check one) A. Joint swelling B. Early morning stiffness C. Reduction of symptoms with high-dose NSAID therapy D. An erythrocyte sedimentation rate ≥60 mm/hr E. Bilateral shoulder or hip stiffness and aching

There must be bilateral shoulder or hip stiffness and aching for at least one month in order to make the diagnosis of polymyalgia rheumatica. Joint swelling occurs occasionally, but neither swelling nor early morning stiffness is necessary to make the diagnosis. Polymyalgia rheumatica does not respond to NSAIDs. The erythrocyte sedimentation rate should be ≥40 mm/hr.

A 47-year-old female presents to your office with a complaint of hair loss. On examination she has a localized 2-cm round area of complete hair loss on the top of her scalp. Further studies do not reveal an underlying metabolic or infectious disorder. Which one of the following is the most appropriate initial treatment? (check one) A. Topical minoxidil (Rogaine) B. Topical immunotherapy C. Intralesional triamcinolone (Kenalog) D. Oral finasteride (Proscar) E. Oral spironolactone (Aldactone)

These findings are consistent with alopecia areata, which is thought to be caused by a localized autoimmune reaction to hair follicles. It occasionally spreads to involve the entire scalp (alopecia totalis) or the entire body (alopecia universalis). Spontaneous recovery usually occurs within 6-12 months, although areas of regrowth may be pigmented differently. Recovery is less likely if the condition persists for longer than a year, worsens, or begins before puberty. The initial treatment of choice for patients older than 10 years of age, in cases where alopecia areata affects less than 50% of the scalp, is intralesional corticosteroid injections. Minoxidil is an alternative for children younger than 10 years of age or for patients in whom alopecia areata affects more than 50% of the scalp. While topical immunotherapy is the most effective treatment for chronic severe alopecia areata, it has the potential for severe side effects and should not be used as a first-line agent. Finasteride inhibits 5 ß-reductase type 2, resulting in a decrease in dihydrotestosterone levels, and is used in the treatment of androgenic alopecia (male-pattern baldness). Similarly, spironolactone is sometimes used for androgenic alopecia because it is an aldosterone antagonist with antiandrogenic effects

A 23-month-old child is brought to your office with a 2-day history of a fever to 102°F (39°C), cough, wheezing, and mildly labored breathing. He has no prior history of similar episodes and there is no improvement with administration of an aerosolized bronchodilator. Which one of the following is now indicated? (check one) A. Bronchodilator aerosol treatment every 6 hours B. Corticosteroids C. An antibiotic D. A decongestant E. Supportive care only

This child has typical findings of bronchiolitis. The initial infection usually occurs by the age of 2 years. It is caused by respiratory syncytial virus (RSV). Bronchodilator treatment may be tried once and discontinued if there is no improvement. Treatment usually consists of supportive care only, including oxygen and intravenous fluids if indicated (SOR B). Corticosteroids, antibiotics, and decongestants are of no benefit. RSV infection may recur, since an infection does not provide immunity. Up to 10% of infected children will have wheezing past age 5, and bronchiolitis may predispose them to asthma.

An 8-year-old female is brought to your office because she has begun to limp. She has had a fever of 38.8°C (101.8°F) and says that it hurts to bear weight on her right leg. She has no history of trauma. On examination, she walks with an antalgic gait and hesitates to bear weight on the leg. Range of motion of the right hip is limited in all directions and is painful. Her sacroiliac joint is not tender, and the psoas sign is negative. Laboratory testing reveals an erythrocyte sedimentation rate of 55 mm/hr (N 0-10), a WBC count of 15,500/mm 3 (N 4500-13,500), and a C-reactiveprotein level of 2.5 mg/dL (N 0.5-1.0). Which one of the following will provide the most useful diagnostic information to further evaluate this patient's problem? (check one) A. MRI B. CT C. A bone scan D. Ultrasonography E. Plain-film radiography

This child meets the criteria for possible septic arthritis. In this case ultrasonography is recommended over other imaging procedures. It is highly sensitive for detecting effusion of the hip joint. If an effusion is present, urgent ultrasound-guided aspiration should be performed. Bone scintigraphy is excellent for evaluating a limping child when the history, physical examination, and radiographic and sonographic findings fail to localize the pathology. CT is indicated when cortical bone must be visualized. MRI provides excellent visualization of joints, soft tissues, cartilage, and medullary bone. It is especially useful for confirming osteomyelitis, delineating the extent of malignancies, identifying stress fractures, and diagnosing early Legg-Calvé-Perthes disease. Plain film radiography is often obtained as an initial imaging modality in any child with a limp. However, films may be normal in patients with septic arthritis, providing a false-negative result.

A primigravida at 38 weeks gestation is concerned that her fetus is getting too large and wants to know what interventions could prevent complications from a large baby. On examination her uterine fundus measures 41 cm from the pubic symphysis. Ultrasonography is performed and an estimated fetal weight of 4000 g (8 lb 13 oz) is reported. Which one of the following management options is supported by the best evidence? (check one) A. Induction of labor B. Cesarean section C. Awaiting spontaneous labor D. Weekly ultrasonography to follow fetal growth

This estimated fetal weight is at the 90th percentile for a term fetus. Unfortunately, the accuracy of fetal weight estimates declines as pregnancy proceeds, and the actual size may be as much as 15% different from the estimate. Delivery of a large infant results in shoulder dystocia more often than delivery of a smaller infant, but most large infants are delivered without complications. Intuitively, it would seem logical to induce labor when the fetus seems to be getting large, but this intervention has been studied in controlled trials and the only difference in outcome was an increase in the cesarean rate for women who underwent elective induction for this indication. Recently, there has been an increase in requests from patients to have an elective cesarean section near term to avoid the risks of labor, including pain, shoulder dystocia, and pelvic relaxation. The American Congress of Obstetricians and Gynecologists (ACOG) recommends consideration of cesarean delivery without a trial of labor if the estimated fetal weight is 4500 g in a mother with diabetes mellitus, or 5000 g in the absence of diabetes. Even at that size, there is not adequate data to show that cesarean section is preferable to a trial of labor. Frequent ultrasonography is often performed to reduce anxiety for both patient and physician, but the problem of accuracy of weight estimates remains an issue even with repeated scans at term.

During rounds, you notice a new rash on a full-term 2-day-old white female. It consists of 1-mm pustules surrounded by a flat area of erythema, and is located on the face, trunk, and upper arms. An examination is otherwise normal, and she does not appear ill. Which one of the following is the most likely diagnosis? (check one) A. Erythema toxicum neonatorum B. Transient neonatal pustular melanosis C. Acne neonatorum D. Systemic herpes simplex E. Staphylococcus aureus sepsis

This infant has the typical "flea-bitten" rash of erythema toxicum neonatorum (ETN). Transient neonatal pustular melanosis is most common in African-American newborns, and the lesions lack the surrounding erythema typical of ETN. Acne neonatorum is associated with closed comedones, mostly on the face. As the infant described is not ill, infectious etiologies are unlikely.

A 45-year-old female presents with a complaint of pain and swelling in her right index finger of 2 days' duration. She reports that 5 days ago she had artificial nails applied, which she removed yesterday due to the pain. She used hydrogen peroxide on the finger, but it did not help. She denies any systemic symptoms or fever. On examination there is erythema and swelling in the lateral nail fold of the right index finger, with purulent material noted. Which one of the following would be the most appropriate treatment for this patient? (check one) A. Removal of the proximal nail fold B. Topical corticosteroids C. Topical antibiotics D. Topical antifungals

This is a common presentation for acute paronychia, which typically is caused by local trauma to the nail fold or cuticle, with resulting inoculation and infection. Topical antibiotics, with or without topical corticosteroids, is one treatment option. Other options include warm compresses, oral antibiotics, and incision and drainage; however, incision and drainage is not always necessary. Removal of the proximal nail fold is used to treat chronic paronychia that is not responsive to other treatments. Topical corticosteroids can be used alone for chronic paronychia, but if used for acute paronychia, they should be combined with antibiotics since acute paronychia is typically caused by a bacterial infection. Topical antifungals are a treatment option for chronic paronychia, which can be associated with a fungal infection, but not for acute paronychia.

You have recently begun caring for a 25-year-old white female who has multiple complaints. You have seen her 3 times for walk-in office visits over the past month. She has shown appreciation for your work during the encounters, but has been critical of your care when talking to the office staff. At times she has been kind and charming, and at other times she has been rude and verbally abusive to your staff. She has a string of multiple relationships in the past, none of which has lasted very long. During times of intense stress, she has sometimes engaged in self-mutilation. She frequently changes jobs and living arrangements. Which one of the following strategies would be most appropriate in the care of this patient? (check one) A. Strive to develop a close relationship with the patient B. Ignore verbal attacks on staff members C. Prescribe lorazepam (Ativan) D. Schedule frequent office visits for follow-up E. Provide detailed, technical explanations for any therapies provided

This patient demonstrates features of borderline personality disorder. These patients often demonstrate instability in interpersonal relationships and self-image, and may be impulsive. They can present with a wide range of symptoms, including depression, anger, paranoia, extreme dependency, self-mutilation, and alternating idealization and devaluation of their physicians. Their lives are often chaotic. Treatment strategies include maintaining a caring but somewhat detached professional stance. A close personal relationship is typically not therapeutic for these patients. Angry outbursts will often have to be tolerated, but limit-setting is necessary with respect to appropriate behaviors. SSRIs, atypical antipsychotics, and mood stabilizers may be of help at times, but anxiolytics are often abused and may be associated with self-mutilating behaviors. These patients tend to respond best to clear, simple, non-technical explanations related to their medical care.

A 50-year-old female presents with a 3-week history of a moderately pruritic rash, characterized by flat-topped violaceous papules 3-4 mm in size. The lesions are located primarily on the volar wrists and forearms, lower legs, and dorsa of both feet. Ten days after the rash first appeared she went to the emergency department and was treated for "possible scabies," but the treatment has made little or no difference. Which one of the following treatments is indicated at this time? (check one) A. Clobetasol (Cormax, Temovate) 0.05% ointment B. Permethrin 5% cream C. Dipyridamole (Persantine) D. Triamcinolone 0.1% cream

This patient has classic lichen planus, with pruritic, symmetrically distributed papular lesions. The violaceous flat-topped papules, usually 3-6 mm in size, are distinct and so characteristic in appearance that a biopsy is usually not necessary to make the diagnosis. First-line treatment is with high-potency topical corticosteroids such as clobetasol, as mid-potency topical agents such as triamcinolone are ineffective. Topical calcineurin inhibitors, including tacrolimus, can be used in cases not responding to topical corticosteroids. While scabies can masquerade as a variety of other dermatoses, retreatment with a scabicide is not indicated in this patient.

A 67-year-old female presents with progressive weakness, dry skin, lethargy, slow speech, and eyelid edema. Of the following medications currently taken by this patient, which one is most likely to be causing her symptoms? (check one) A. Donepezil (Aricept) B. Lithium C. Lisinopril (Prinivil, Zestril) D. Alendronate (Fosamax) E. Glyburide (DiaBeta, Micronase)

This patient has classic signs of hypothyroidism. Of the drugs listed, only lithium is associated with the development of hypothyroidism. In patients taking lithium, it is recommended that in addition to regular serum lithium levels, thyroid function tests including total free T4, and TSH be obtained yearly. ref: Griswold KS, Pessar LF: Management of bipolar disorder.

A 36-year-old male complains of clear rhinorrhea, nasal congestion, and watery, itchy eyes for several months. Tests in the past have suggested that he has an allergy to dust mites. Which one of the following is most likely to provide the most relief from his symptoms? (check one) A. Oral antihistamines B. An oral leukotriene-receptor antagonist C. Intranasal antihistamines D. Intranasal corticosteroids E. Furnace filters and mite-proof bedding covers

This patient has classic symptoms of allergic rhinitis. Intranasal corticosteroids are considered the mainstay of treatment for mild to moderate cases. In multiple studies, intranasal corticosteroid sprays have proven to be more efficacious than the other options listed, even for ocular symptoms. Air filtration systems and bedding covers have not been shown to reduce symptoms.

A 34-year-old female who delivered a healthy infant 18 months ago complains of a milky discharge from both nipples. She reports that normal periods have resumed since cessation of breastfeeding 6 months ago. She takes ethinyl estradiol/norgestimate (Ortho Tri-Cyclen) for birth control. A complete review of systems is otherwise negative. The most likely cause of the discharge is (check one) A. a medication side effect B. breast cancer C. a hypothalamic tumor D. hypothyroidism

This patient has galactorrhea, which is defined as a milk-like discharge from the breast in the absence of pregnancy in a non-breastfeeding patient who is more than 6 months post partum. It is more common in women ages 20-35 and in women who are previously parous. It also can occur in men. Medication side effect is the most common etiology. The most common pharmacologic cause of galactorrhea is oral contraceptives. Oral contraceptives that contain estrogen can both suppress prolactin inhibitory factor and stimulate the pituitary directly, both of which can cause galactorrhea. Other medications that can cause galactorrhea include metoclopramide, cimetidine, risperidone, methyldopa, codeine, morphine, verapamil, SSRIs, butyrophenones, dopamine-receptor blockers, tricyclics, phenothiazines, and thioxanthenes. Breast cancer is unlikely to present with a bilateral milky discharge. The nipple discharge associated with cancer is usually unilateral and bloody. Pituitary tumors are a pathologic cause of galactorrhea due to the hyperprolactinemia that is caused by the blockage of dopamine from the hypothalamus, or by the direct production of prolactin. However, patients often have symptoms such as headache, visual disturbances, temperature intolerance, seizures, disordered appetite, polyuria, and polydipsia. Patients with prolactinomas often have associated amenorrhea. These tumors are associated with marked levels of serum prolactin, often >200 ng/mL. Hypothalamic lesions such as craniopharyngioma, primary hypothalamic tumor, metastatic tumor, histiocytosis X, tuberculosis, sarcoidosis, and empty sella syndrome are significant but infrequent causes of galactorrhea, and generally cause symptoms similar to those of pituitary tumors, particularly headache and visual disturbances. It is rare for primary hypothyroidism to cause galactorrhea in adults. Symptoms that would be a clue to this diagnosis include fatigue, constipation, menstrual irregularity, weight changes, and cold intolerance.

A 30-year-old male complains of the gradual onset of anterior right knee pain on climbing the stairs. On examination there is no effusion, but there is tenderness over the medial retinaculum. There is good ligament strength, and range of motion is normal. When the knee is extended from 90° flexion to full extension, the patella deviates laterally. Which one of the following would be the best initial treatment for this condition? (check one) A. Bracing B. Taping C. NSAIDs D. Arthroscopic surgery E. Physical therapy

This patient has patellofemoral stress syndrome. It is often called runner's knee or anterior knee pain. The patellofemoral joint comprises the patella and femoral trochlea. The best initial treatment is physical therapy. Bracing, taping, and medications are unlikely to have better outcomes. Arthroscopic surgery is not indicated.

A previously healthy 60-year-old male is diagnosed with multiple myeloma after a workup for an incidental finding on routine laboratory work. He has no identified organ or tissue damage and is asymptomatic. Which one of the following would be appropriate treatment of this patient's condition? (check one) A. No treatment B. Chemotherapy C. Autologous stem cell transplantation D. Radiation

This patient has smoldering (asymptomatic) multiple myeloma. He does not have any organ or tissue damage related to this disease and has no symptoms. Early treatment of these patients does not improve mortality (SOR A) and may increase the likelihood of developing acute leukemia. The standard treatment for symptomatic patients under age 65 is autologous stem cell transplantation. Patients over 65 who are healthy enough to undergo transplantation would also be appropriate candidates. Patients who are not candidates for autologous stem cell transplantation generally receive melphalan and prednisolone with or without thalidomide. Radiotherapy can be used to relieve metastatic bone pain or spinal cord compression.

A 72-year-old male has had persistent interscapular pain with movement since rebuilding his deck 1 week ago. He rates the pain as 6 on a 10-point scale. A chest radiograph shows a thoracic vertebral compression fracture. Which one of the following would be most appropriate at this point? (check one) A. Complete bed rest for 2 weeks B. Markedly decreased activity until the pain lessens, and follow-up in 1 week C. Referral for vertebroplasty as soon as possible D. NSAIDs and referral for physical therapy

This patient has suffered a thoracic vertebral compression fracture. Most can be managed conservatively with decreased activity until the pain is tolerable, possibly followed by some bracing. Vertebroplasty is an option when the pain is not improved in 2 weeks. Complete bed rest is unnecessary and could lead to complications. Physical therapy is not indicated, and NSAIDs should be used with caution.

You see a 6-year-old male for the third time in 3 months with a persistently painful hand condition. He has been treated with oral amoxicillin, followed by oral trimethoprim/ sulfamethoxazole (Bactrim, Septra), with no improvement. A physical examination reveals retraction of the proximal nail fold, absence of the cuticle, and erythema and tenderness around the nail fold area. The thumb and second and third fingers are affected on both hands. The patient is otherwise healthy. First-line treatment for this condition includes: (check one) A. warm soaks three times a day B. avoidance of emollient lotions C. a topical corticosteroid cream D. an oral antifungal agent

This patient has symptoms and signs consistent with chronic paronychia. This condition is often associated with chronic immersion in water, contact with soaps or detergents, use of certain systemic drugs (antiretrovirals, retinoids) and, as is most likely in a 6-year-old child, finger sucking. Findings on examination are similar to those of acute paronychia, with tenderness, erythema, swelling, and retraction of the proximal nail fold. Often the adjacent cuticle is absent. Chronic paronychia has usually been persistent for at least 6 weeks by the time of diagnosis. In addition to medication, basic treatment principles for the condition include avoidance of contact irritants, avoiding immersion of the hands in water, and use of an emollient. Topical corticosteroids have higher efficacy for treating chronic paronychia compared to oral antifungals (SOR B), particularly given the young age of the patient. A topical antifungal can also be tried in conjunction with the corticosteroid.

A 72-year-old African-American male with New York Heart Association Class III heart failure sees you for follow-up. He has shortness of breath with minimal exertion. The patient is adherent to his medication regimen. His current medications include lisinopril (Prinivil, Zestril), 40 mg twice daily; carvedilol (Coreg), 25 mg twice daily; and furosemide (Lasix), 80 mg daily. His blood pressure is 100/60 mm Hg, and his pulse rate is 68 beats/min and regular. Findings include a few scattered bibasilar rales on examination of the lungs, an S3 gallop on examination of the heart, and no edema on examination of the legs. An EKG reveals a left bundle branch block, and echocardiography reveals an ejection fraction of 25%, but no other abnormalities. Which one of the following would be most appropriate at this time? (check one) A. Increase the lisinopril dosage to 80 mg twice daily B. Increase the carvedilol dosage to 50 mg twice daily C. Increase the furosemide dosage to 160 mg daily D. Refer for coronary angiography E. Refer for cardiac resynchronization therapy

This patient is already receiving maximal medical therapy. The 2002 joint guidelines of the American College of Cardiology, the American Heart Association (AHA), and the North American Society of Pacing and Electrophysiology endorse the use of cardiac resynchronization therapy (CRT) in patients with medically refractory, symptomatic, New York Heart Association (NYHA) class III or IV disease with a QRS interval of at least 130 msec, a left ventricular end-diastolic diameter of at least 55 mm, and a left ventricular ejection fraction (LVEF) ≤30%. Using a pacemaker-like device, CRT aims to get both ventricles contracting simultaneously, overcoming the delayed contraction of the left ventricle caused by the left bundle-branch block. These guidelines were refined by an April 2005 AHA Science Advisory, which stated that optimal candidates for CRT have a dilated cardiomyopathy on an ischemic or nonischemic basis, an LVEF ≤0.35, a QRS complex ≥120 msec, and sinus rhythm, and are NYHA functional class III or IV despite maximal medical therapy for heart failure.

A 23-year-old gravida 1 para 0 at 36 weeks gestation presents to the office complaining of ankle swelling and headache for the past 2 days. She denies any abdominal pain or visual disturbances. On examination you note a fundal height of 35 cm, a fetal heart rate of 140 beats/min, 2+ lower extremity edema, and a blood pressure of 144/92 mm Hg. A urine dipstick shows 1+ proteinuria. A cervical examination reveals 2 cm dilation, 90% effacement, -1 station, and vertex presentation. Which one of the following is the most appropriate next step in the management of this patient? (check one) A. Laboratory evaluation, fetal testing, and 24-hour urine for total protein B. Ultrasonography to check for fetal intrauterine growth restriction C. Initiation of antihypertensive treatment D. Immediate induction of labor E. Immediate cesarean delivery

This patient most likely has preeclampsia, which is defined as an elevated blood pressure and proteinuria after 20 weeks gestation. The patient needs further evaluation, including a 24-hour urine for quantitative measurement of protein, blood pressure monitoring, and laboratory evaluation that includes hemoglobin, hematocrit, a platelet count, and serum levels of transaminase, creatinine, albumin, LDH, and uric acid. A peripheral smear and coagulation profiles also may be obtained. Antepartum fetal testing, such as a nonstress test to assess fetal well-being, would also be appropriate. Ultrasonography should be done to assess for fetal intrauterine growth restriction, but only after an initial laboratory and fetal evaluation. Delivery is the definitive treatment for preeclampsia. The timing of delivery is determined by the gestational age of the fetus and the severity of preeclampsia in the mother. Vaginal delivery is preferred over cesarean delivery, if possible, in patients with preeclampsia. It is not necessary to start this patient on antihypertensive therapy at this point. An obstetric consultation should be considered for patients with preeclampsia.

An 8-year-old male is brought to your office for evaluation of recurrent headaches. His mother explains that the headaches occur at least twice a week and often require him to miss school. The patient says he sometimes feels nauseated and that being in a dark room helps. His mother states that she had migraines as a child. The child's only other medical issue is constipation. A head CT ordered by another physician was negative. Which one of the following would be best for preventing these episodes? (check one) A. Sumatriptan (Imitrex) B. Ibuprofen C. Carbamazepine (Tegretol) D. Propranolol (Inderal) E. Amitriptyline

This patient most likely is suffering from recurrent migraine headaches; at the described frequency and intensity, he meets the criteria for prophylactic medication. Ibuprofen or acetaminophen could still be used as rescue medications, but a daily agent is indicated and propranolol is the best choice for this patient (SOR B). Sumatriptan is not approved for children under the age of 12 years. Carbamazepine has significant side effects and requires monitoring. Amitriptyline is a commonly used agent, but it could worsen his constipation.

A 7-year-old female with a history of asthma is brought to your office for a routine follow-up visit. She has a history of exercise-induced asthma, but also has had exacerbations in the past that were unrelated to exercise. In the past month, she has premedicated herself with albuterol (Proventil, Ventolin) with a spacer before recess 5 days/week as usual. She has also needed her albuterol to treat symptoms (wheezing and/or shortness of breath) once or twice per week and had one exacerbation requiring medical treatment in the past year. She has had no nighttime symptoms. Albuterol as needed is her only medication. After reinforcing asthma education, which one of the following would be most appropriate? (check one) A. Referral to an asthma specialist B. Addition of a low-dose inhaled corticosteroid C. Addition of a long-acting β-agonist D. Elimination of premedication with albuterol, restricting use to an as-needed basis E. No changes to her regimen

This patient's asthma is well-controlled according to the 2007 NHLBI asthma guidelines. The "rule of twos" is useful in assessing asthma control: in children under the age of 12, asthma is NOT well-controlled if they have had symptoms or used a β-agonist for symptom relief more than twice per week, had two or more nocturnal awakenings due to asthma symptoms in the past month, or had two or more exacerbations requiring systemic corticosteroids in the past year. For individuals over 12 years of age, there must be more than two nocturnal awakenings per month to classify their asthma as not well controlled. Exercise-induced asthma is considered separately. A β-agonist used as premedication before exercise is not a factor when assessing asthma control. Since this patient does not exceed the rule of twos, her asthma is categorized as well-controlled and no changes to her therapy are indicated. Asthma education should be reinforced at every visit.

Systolic vs. Diastolic HF

This patient's history and clinical examination suggest heart failure. The most important distinction to make is whether it is diastolic or systolic, as the drug treatment may be somewhat different. Physical findings and chest radiographs do not distinguish systolic from diastolic heart failure. An echocardiogram is the study of choice, as it will assess left ventricular function. In diastolic dysfunction, the left ventricular ejection fraction is normal or slightly elevated. Diastolic failure is more common in elderly females and patients with hypertension, and less common in patients with a previous history of coronary artery disease. Diuretics and angiotensin receptor blockers (ARBs) are useful treatments. Because of their effects on diastolic filling times, tachycardia and atrial fibrillation often cause decompensation in patients with diastolic heart failure. At this time, cardiac catheterization is not indicated, and a stress test will not provide useful information. If the patient had systolic failure, a workup for ischemic disease would be needed, but most cases of diastolic dysfunction are not caused by ischemia. While hyperthyroidism can cause tachycardia and atrial fibrillation, the more immediate issue in this patient is the heart failure, which requires diagnosis and treatment. A pulmonary embolus can cause shortness of breath but usually has an acute onset, so a D-dimer level would not help at this time.

A 21-year-old primigravida at 28 weeks gestation complains of the recent onset of itching. On examination she has no obvious rash. The pruritus started on her palms and soles and spread to the rest of her body. Laboratory evaluation reveals elevated serum bile acids and mildly elevated bilirubin and liver enzymes. The most effective treatment for this condition is: (check one) A. triamcinolone (Kenalog) cream B. cholestyramine (Questran) C. diphenhydramine (Benadryl) D. doxylamine succinate E. ursodiol (Actigall)

This patient's symptoms and laboratory values are most consistent with intrahepatic cholestasis of pregnancy. Ursodiol has been shown to be highly effective in controlling the pruritus and decreased liver function (SOR A) and is safe for mother and fetus. Topical antipruritics and oral antihistamines are not very effective. Cholestyramine may be effective in mild or moderate intrahepatic cholestasis, but is less effective and safe than ursodiol.

Contraindications to thrombolytic therapy in acute stroke include which one of the following? (check one) A. Age >80 B. Resolving transient ischemic attack C. Blood glucose >200 mg/dL D. Deficit present for >1 hour

Thrombolysis is now an approved treatment for acute stroke. The critical time frame is 3 hours after the onset of the deficit. Beyond that time span, the use of thrombolytic agents is contraindicated. Advanced age per se is not a contraindication to thrombolytic therapy. Contraindications include blood glucose levels <50 mg/dL or >400 mg/dL, resolving transient ischemic attack, and hemorrhage visible on a CT scan

Which one of the following is true concerning postpartum depression? (check one) A. It has no effect on cognitive development of the child B. It is directly related to the desired gender of the infant C. It is usually transient, lasting about 10 days D. Thyroid function should always be assessed in women with postpartum depression

Thyroid function must be evaluated in women with postpartum depression since both hyperthyroidism and hypothyroidism are more common post partum. Postpartum depression may impair cognitive and behavioral development in the child. It is not related to the desired gender of the child, breastfeeding, or education level of the mother. It should be differentiated from the short-term "baby blues" that resolve within about 10 days. Sertraline is considered first-line treatment for postpartum depression in women who are breastfeeding.

An 8-year-old male presents to your office 2 days after returning from a trip to Mexico with his family. He developed watery, nonbloody diarrhea on the day of departure. He has mild abdominal cramping, but no fever or vomiting. His mother had similar symptoms, which were milder and resolved with over-the-counter treatments. Which one of the following would be most appropriate to treat this patient's condition? (check one) A. Metronidazole (Flagyl) B. Ciprofloxacin (Cipro) C. Azithromycin (Zithromax) D. Mebendazole E. Metoclopramide (Reglan)

Traveler's diarrhea commonly occurs in travelers to Mexico and developing countries. It is usually caused by bacterial organisms such as Escherichia coli, Campylobacter, Shigella, and Salmonella. Viral and parasitic organisms are less common causes, unless the diarrhea persists for 2 weeks. Appropriate medications include antidiarrheal agents such as loperamide, bismuth subsalicylate, and antibiotics. Fluoroquinolones are effective in adults, but should not be used in an 8-year-old. Azithromycin isgenerally effective and safe in children. Metronidazole, mebendazole, and metoclopramide would not be likely to successfully treat bacterial traveler's diarrhera.

A 75-year-old male consults you after his family expresses concern about his loss of interest in his usual activities. They believe he has become increasingly withdrawn since the death of his wife 8 months earlier. You note he has lost 8 kg (18 lb) since his last office visit 6 months earlier. He does not drink alcohol. His physical examination is unremarkable for his age except for a blood pressure of 105/70 mm Hg. Detailed laboratory studies, including thyroid function tests, are all within normal limits. He tells you he would be fine if he could just get some sleep. His Mini-Mental State Examination is normal, but he is obviously clinically depressed. The most appropriate medication for his depression would be: (check one) A. trazodone (Oleptro) B. mirtazapine (Remeron) C. bupropion (Wellbutrin) D. amitriptyline E. nortriptyline (Pamelor)

Trazodone may be useful for insomnia, but is not recommended as a primary antidepressant because it causes sedation and orthostatic hypotension at therapeutic doses. Bupropion would aggravate this patient's insomnia. Tricyclic antidepressants may be effective, but are no longer considered first-line treatments because of side effects and because they can be cardiotoxic. Mirtazapine has serotonergic and noradrenergic properties and is associated with increased appetite and weight gain. It may be particularly useful for patients with insomnia and weight loss.

You have been treating a 43-year-old male for unipolar depression for 4 years. He has developed treatment-resistant depression, and despite having a good initial response to an SSRI, his symptoms are worsening. He has failed to improve despite escalated doses of multiple SSRIs and SNRIs. He is currently taking citalopram (Celexa), 60 mg daily. Of the following, the most effective adjunctive therapy would be augmentation with: (check one) A. lithium bicarbonate B. high-dose triiodothyronine C. an atypical antipsychotic, such as olanzapine (Zyprexa) D. an anticonvulsant, such as gabapentin (Neurontin)

Up to one-third of patients with unipolar depression will fail to respond to treatment with a single antidepressant, despite adequate dosing and an appropriate treatment interval. Lithium, triiodothyronine (T3 ), and atypical antipsychotics can all provide clinical improvement when used in conjunction with the ineffective antidepressant. The American Psychiatric Association and the Institute for Clinical Systems Improvement both recommend a trial of lithium or low-dose T 3 for patients who have an incomplete response to antidepressant therapy. A meta-analysis showed that a serum lithium level ≥0.5 mEq/L and a treatment duration of 2 weeks or greater resulted in a good response (SOR A). While thyroid supplementation as adjunctive therapy is effective, the recommended dosage is no higher than 50 μg/day (SOR B). Atypical antipsychotics can be used as add-on therapy, but are not as effective as lithium or T3 (SOR B). Anticonvulsant medications such as gabapentin have been shown to be effective in the management of bipolar affective disorder, but not as adjunctive therapy in the treatment of unipolar depression resistant to single-agent antidepressants.

A 21-year-old female complains of bulging veins in her right shoulder region, along with swelling and a "tingling" sensation in her right arm that has developed over the past 2 days. There were no unusual events other than her regular workouts with her swim team. Ultrasonography confirms an upper extremity deep-vein thrombosis of her right axillary vein. Which one of the following would be the most appropriate treatment? (check one) A. Intravenous heparin for 72 hours, followed by oral warfarin (Coumadin) for 3 months B. Low molecular weight heparin (LMWH) subcutaneously for 5 days only C. LMWH subcutaneously for at least 5 days, followed by oral warfarin for 3 months D. LMWH subcutaneously for at least 5 days, followed by oral warfarin indefinitely E. Oral warfarin for 3 months

Upper extremity deep-vein thrombosis (UE-DVT) accounts for 4% of all cases of DVT. Catheter-related thromboses make up the majority of these cases. Occult cancer, use of oral contraceptives, and inheritable thrombophilia are other common explanations. Another proposed risk factor is the repetitive compression of the axillary-subclavian vein in athletes or laborers, which is the most likely cause of this patient's UE-DVT. Taken as a whole, UE-DVT is generally associated with fewer venous complications, including less chance for thromboembolism, postphlebitic syndrome, and recurrence compared to lower-extremity deep-vein thrombosis (LE-DVT). However, the rates of these complications are still high enough that most experts recommend treatment identical to that of LE-DVT. Specifically, heparin should be given for 5 days, and an oral vitamin-K antagonist for at least 3 months.

You see a 78-year-old male in the hospital the day after his hip-replacement surgery. He has not voided in the past 12 hours. A urethral catheter is placed and 500 mL of urine is removed from his bladder. Which one of the following is most likely to improve the success rate of a voiding trial? (check one) A. Using a specialized catheter coudé instead of a standard catheter B. Leaving the catheter in place for at least 2 weeks C. Immediately removing the catheter to prevent a urinary tract infection D. Starting tamsulosin (Flomax), 0.4 mg daily, at the time of catheter insertion E. Starting antibiotic prophylaxis at the time of catheter insertion

Urinary retention is a common problem in hospitalized patients, especially following certain types of surgery. Starting an a-blocker at the time of insertion of the urethral catheter has been shown to increase the success of a voiding trial (SOR A). Voiding trial success rates have not been shown to be improved by leaving the catheter in for 2 weeks, immediate removal of the catheter, using a specialized catheter, or antibiotic prophylaxis.

HBsAg Post Exposure Prophylaxis: Vaccinated vs. Unvaccianted

VACCINATED - Test for HBsAg first UNVACCINATED - HBIG + Vaccine

A 52-year-old male requests "everything you've got" to help him stop smoking. You review common barriers to quitting and the benefits of cessation with him, and develop a plan that includes follow-up. He chooses to start varenicline (Chantix) to assist with his efforts, and asks about also using nicotine replacement. Which one of the following would be accurate advice? (check one) A. Combining these medications has not proven to be beneficial B. The addition of transdermal nicotine, but not nicotine gum, has proven benefits C. The combination is highly efficacious D. Nicotine replacement doses need to be doubled in a patient taking varenicline E. The combination of nicotine and varenicline is potentially lethal

Varenicline works by binding to nicotine receptors in the brain, providing much lower stimulation than nicotine itself would. This has the effect of reducing the reinforcement and reward that smoking provides to the brain. However, this medication also blocks the benefit a patient would receive from nicotine replacement products. Studies have shown that using nicotine replacement products concurrently with varenicline leads to an increase in nausea, headaches, dizziness, and fatigue.

A 65-year-old male who has been in good health presents to your office with a 2-day history of a sensation of pressure and hearing loss in his left ear. A physical examination and a thorough neurologic examination are both unremarkable. Both tympanic membranes are normal. An audiogram shows a 30-decibel hearing loss at three consecutive frequencies in the left ear, with normal hearing on the right. Placing a vibrating tuning fork in the midline of the forehead reveals sound lateralizing to the right ear. Which one of the following would be most appropriate at this point? (check one) A. CT B. A CBC, metabolic profile, and thyroid studies C. Nifedipine (Procardia) D. Acyclovir (Zovirax) E. Oral corticosteroids

When a patient presents with sudden hearing loss it is important to distinguish between sensorineural and conductive hearing loss. Patients should be asked about previous episodes, and the workup should include both an assessment for bilateral hearing loss and a neurologic examination. Sudden sensorineural hearing loss is diagnosed by audiometry demonstrating a 30-decibel hearing loss at three consecutive frequencies, with no other cause indicated from the physical examination. Evaluation for retrocochlear pathology may include auditory brainstem response, MRI, or follow-up audiometry. Routinely prescribing antiviral agents, thrombolytics, vasodilators, vasoactive substances, or antioxidants is not recommended. Oral corticosteroids may be offered as initial therapy, and hyperbaric oxygen therapy may be helpful within 3 months of diagnosis. The guidelines also strongly recommend against routine laboratory tests or CT of the head as part of the initial evaluation.

A 4-year-old is brought to the emergency department with abdominal pain and is noted to have 3+ proteinuria on a dipstick. Three days later the pain has resolved spontaneously, and a repeat urinalysis in your office shows 2+ proteinuria with normal findings on microscopic examination. A metabolic panel, including creatinine and total protein, is also normal. Which one of the following would be most appropriate at this point? (check one) A. Renal ultrasonography B. A spot first morning urine protein/creatinine ratio C. An antinuclear antibody and complement panel D. Referral to a nephrologist

When proteinuria is noted on a dipstick and the history, examination, full urinalysis, and serum studies suggest no obvious underlying problem or renal insufficiency, a urine protein/creatinine ratio is recommended. This test correlates well with 24-hour urine protein, which is particularly difficult to collect in a younger patient. Renal ultrasonography is appropriate once renal insufficiency or nephritis is established. If pathogenic proteinuria is confirmed, an antinuclear antibody and/or complement panel may be indicated. A nephrology referral is not necessary until the presence of kidney disease or proteinuria from a cause other than benign postural proteinuria is confirmed.

A 75-year-old male has not seen a physician in 25 years and presents with advanced Parkinson's disease. The best initial treatment would be: (check one) A. Referral to a neurosurgeon for thalamotomy B. Amantadine (Symmetrel) C. Benztropine (Cogentin) D. Pramipexole (Mirapex) E. Carbidopa/levodopa (Sinemet)

While anticholinergics such as benztropine and amantadine may provide some improvement of symptoms, these effects wane within a few months. Such medications are not a good option in this patient with advanced disease. Dopamine agonists provide some improvement in motor complications, but are mainly used to delay the introduction of levodopa in younger patients, to avoid levodopa-related adverse reactions. Carbidopa/levodopa is better for initial therapy in older patients, and those who present with more severe symptoms. Slow-release versions of this combination may decrease motor fluctuations. Stereotactic thalamotomy is used to ameliorate tremors that have become disabling. This procedure has been replaced by other surgical options such as pallidotomy and high-frequency, deep-brain stimulation of specific nuclei

Leukoplakia

White spot on mucus membranes that stems from chemical irritation and can progress to SCC

An asymptomatic 55-year-old male visits a health fair, where he has a panel of blood tests done. He brings the results to you because he is concerned about the TSH level of 12.0 µU/mL (N 0.45-4.5). His free T4 level is normal. Which one of the following is most likely to be associated with this finding? (check one) A. Atrial fibrillation B. Reduced bone density C. Systolic heart failure D. Elevated LDL cholesterol E. Type 2 diabetes mellitus

With subclinical thyroid dysfunction, TSH is either below or above the normal range, free T3 or T4 levels are normal, and the patient has no symptoms of thyroid disease. Subclinical hypothyroidism (TSH >10 µU/mL) is likely to progress to overt hypothyroidism, and is associated with increased LDL cholesterol. Subclinical hyperthyroidism (TSH <0.1 µU/mL) is associated with the development of atrial fibrillation, decreased bone density, and cardiac dysfunction. Neither type of subclinical thyroid dysfunction is associated with diabetes mellitus. There is insufficient evidence of benefit to warrant early treatment of either condition. Ref: Wilson GR, Curry RW Jr: Subclinical thyroid disease

A 19-year-old white female presents for an initial family planning evaluation. Specifically, she is interested in oral contraception. She is not presently sexually active, but has a steady boyfriend. She has no contraindications to oral contraceptive use. She has mild acne vulgaris. You discuss possible side effects and benefits of combined oral contraceptives, including improvement of her acne. Which one of the following is also associated with oral contraceptive use? (check one) A. Increased risk of ovarian cancer B. Decreased risk of ovarian cysts C. Increased risk for ectopic pregnancy D. Increased incidence of dysmenorrhea

Women who take combination oral contraceptives have a reduced risk of both ovarian and endometrial cancer. This benefit is detectable within a year of use and appears to persist for years after discontinuation. Other benefits include a reduction in dysfunctional uterine bleeding and dysmenorrhea; a lower incidence of ovarian cysts, ectopic pregnancy and benign breast disease; and an increase in hemoglobin concentration. Many women also benefit from the convenience of menstrual regularity. All combination oral contraceptives raise sex hormone-binding globulin and decrease free testosterone concentrations, which can lead to improvement in acne

A 72-year-old white male in otherwise good health complains of generalized pruritus that worsens in the winter. The itching is most intense after he bathes. He recently noticed a rash on his abdomen and legs as well. On examination you note poorly defined red, scaly plaques with fine fissures on the abdomen. No eruption is present at other pruritic sites. Which one of the following is the most likely cause of this problem? (check one) A. Stasis dermatitis B. Lichen simplex chronicus C. Xerosis D. Rosacea E. Candidiasis

Xerosis is a pathologic dryness of the skin that is especially prominent in the elderly. It is probably caused by minor abnormalities in maturation of the epidermis that lead to decreased hydration of the superficial portion of the stratum corneum. Xerosis often intensifies in winter, because of the lower humidity and cold temperatures. Stasis dermatitis, due to chronic venous insufficiency, appears as a reddish-brown discoloration of the lower leg. Lichen simplex chronicus, the end result of habitual scratching or rubbing, usually presents as isolated hyperpigmented, edematous lesions, which become scaly and thickened in the center. Rosacea is most often seen on the face as an erythematous, acneiform eruption, which flushes easily and is surrounded by telangiectasia. Candidiasis is an opportunistic infection favoring areas that are warm, moist, and macerated, such as the perianal and inguinal folds, inframammary folds, axillae, interdigital areas, and corners of the mouth

Is edema normal in pregnancy

Yes, not part of criteria for pre-eclampsia

You see a 23-year-old gravida 1 para 0 for her prenatal checkup at 38 weeks gestation. She complains of severe headaches and epigastric pain. She has had an uneventful pregnancy to date and had a normal prenatal examination 2 weeks ago. Her blood pressure is 140/100 mm Hg. A urinalysis shows 2+ protein; she has gained 5 lb in the last week, and has 2+ pitting edema of her legs. The most appropriate management at this point would be: (check one) A. Strict bed rest at home and reexamination within 48 hours B. Admitting the patient to the hospital for bed rest and frequent monitoring of blood pressure, weight, and proteinuria C. Admitting the patient to the hospital for bed rest and monitoring, and beginning hydralazine (Apresoline) to maintain blood pressure below 140/90 mm Hg D. Admitting the patient to the hospital, treating with parenteral magnesium sulfate, and planning prompt delivery either vaginally or by cesarean section

`This patient manifests a rapid onset of preeclampsia at term. The symptoms of epigastric pain and headache categorize her preeclampsia as severe. These symptoms indicate that the process is well advanced and that convulsions are imminent. Treatment should focus on rapid control of symptoms and delivery of the infant.

Which one of the following sets of additional findings would indicate that the patient suffers from severe anorexia nervosa? (check one) A. Hypertension, tachycardia, and hyperthermia B. Hypertension, tachycardia, and hypothermia C. Hypotension, tachycardia, and hypothermia D. Hypotension, bradycardia, and hyperthermia E. Hypotension, bradycardia, and hypothermia

all low

Schizophrenia Dx

delusions, hallucinations, disorganized speech, grossly disorganized or catatonic behavior, and negative symptoms (i.e., affective flattening, alogia, or avolition).

Which one of the following historical features is most suggestive of congestive heart failure in a 6-month-old white male presenting with tachypnea? (check one) A. Diaphoresis with feeding B. Fever C. Nasal congestion D. Noisy respiration or wheezing E. Staccato cough

ymptoms of congestive heart failure in infants are often related to feedings. Only small feedings may be tolerated, and dyspnea may develop with feedings. Profuse perspiration with feedings, is characteristic, and related to adrenergic drive. Older children may have symptoms more similar to adults, but the infant's greatest exertion is related to feeding. Fever and nasal congestion are more suggestive of infectious problems. Noisy respiration or wheezing does not distinguish between congestive heart failure, asthma, and infectious processes. A staccato cough is more suggestive of an infectious process, including pertussis.


Conjuntos de estudio relacionados

Molecule 3 Chapter 16-18 questions

View Set

Chapter 13 Molecular Microbiology

View Set

Chapter 38 Gastrointestinal Assessment 2

View Set

government: chapter 1-3 final test review

View Set

TEST_3_18_Practical Applications: Response Biases

View Set

Econ 102 Frank Fossen Review for Final

View Set